You are on page 1of 91

TM

Step 3

Content Description and


Sample Test Materials

2005
A Joint Program of the Federation of State
Medical Boards of the United States, Inc., and
the National Board of Medical Examiners®
Copyright © 2004 by the Federation of State Medical Boards of the United States, Inc. and the National
Board of Medical Examiners® (NBME®). All rights reserved. The USMLE™ is a joint program of the
Federation of State Medical Boards of the United States, Inc. and the National Board of Medical Examiners.
CONTENTS

Introduction . . . . . . . . . . . . . . . . . . . . . . . . . . . . . . . . . . . . . . . . . . . . . . . . . . 2
Preparing for the Test, Applying for the Test,
Scheduling Test Dates, and Testing . . . . . . . . . . . . . . . . . . . . . . . . . 2
Examination Format . . . . . . . . . . . . . . . . . . . . . . . . . . . . . . . . . . . . . . . . . . . 2
Multiple-choice Items . . . . . . . . . . . . . . . . . . . . . . . . . . . . . . . . . . . 3
Primum Computer-based Case Simulations . . . . . . . . . . . . . . . . . . . 3
Purpose of the Examination . . . . . . . . . . . . . . . . . . . . . . . . . . . . . . . . . . . . . . 3
Examination Design . . . . . . . . . . . . . . . . . . . . . . . . . . . . . . . . . . . . . . . . . . . 4
Clinical Context of Step 3 . . . . . . . . . . . . . . . . . . . . . . . . . . . . . . . . 4
Clinical Settings . . . . . . . . . . . . . . . . . . . . . . . . . . . . . . . . . . . . . . . 5
Content Description . . . . . . . . . . . . . . . . . . . . . . . . . . . . . . . . . . . . . . . . . . . . 6
Figure 1: Step 3 Clinical Encounter Frames . . . . . . . . . . . . . . . . . . . 6
Figure 2: Step 3 Physician Tasks . . . . . . . . . . . . . . . . . . . . . . . . . . . 7
Figure 3: Step 3 Blueprint . . . . . . . . . . . . . . . . . . . . . . . . . . . . . . . . 8
Problem/Disease List . . . . . . . . . . . . . . . . . . . . . . . . . . . . . . . . . . . . 9
Evaluative Objectives . . . . . . . . . . . . . . . . . . . . . . . . . . . . . . . . . . . 19
Step 3 Test Question Formats . . . . . . . . . . . . . . . . . . . . . . . . . . . . . . . . . . . 23
One Best Answer Questions . . . . . . . . . . . . . . . . . . . . . . . . . . . . . . 23
Primum Computer-based Case Simulations . . . . . . . . . . . . . . . . . . 25
Introduction . . . . . . . . . . . . . . . . . . . . . . . . . . . . . . . . . . . . . . 25
Description . . . . . . . . . . . . . . . . . . . . . . . . . . . . . . . . . . . . . . . 25
Case Interface and Format . . . . . . . . . . . . . . . . . . . . . . . . . . . 25
The Patient . . . . . . . . . . . . . . . . . . . . . . . . . . . . . . . . . . . . . . . 26
The Healthcare Network and Facility . . . . . . . . . . . . . . . . . . . 26
Evaluative Objectives and Assessment of Your Performance . . 27
Responsibilities of the Physician . . . . . . . . . . . . . . . . . . . . . . . 27
Frequently Asked Questions . . . . . . . . . . . . . . . . . . . . . . . . . . 29
Sample Step 3 Questions . . . . . . . . . . . . . . . . . . . . . . . . . . . . . . . . . . . . . . . 35
USMLE Step 3 Laboratory Values . . . . . . . . . . . . . . . . . . . . . . . . . 36
Answer Form for Step 3 Sample Questions . . . . . . . . . . . . . . . . . . 38
Sample Step 3 Questions . . . . . . . . . . . . . . . . . . . . . . . . . . . . . . . . 39
Answer Key for Sample Questions . . . . . . . . . . . . . . . . . . . . . . . . . 90
Introduction The Step 3 examination consists of questions ("test
items") presented in standard multiple-choice formats,
This booklet is intended to help you prepare for Step 3 as described on pages 23–24 of this booklet, and
of the United States Medical Licensing Examination Primum CCS, a test format that allows you to provide
(USMLE) if you are an applicant with an eligibility care for a simulated patient, as described on pages
period that has an ending date in 2005. Eligibility 25B34. The test items are divided into "blocks" (see
periods are explained on pages 15B16 of the 2005 Test Lengths and Formats on page 12 of the Bulletin),
USMLE Bulletin of Information, with which you must and test item formats may vary within each block. You
become familiar to apply for the examination. In may want to study the descriptions of test item formats
addition to reading the Bulletin, you should run the that follow before you run the sample test items. A
sample Step 3 test materials and tutorials provided at Normal Laboratory Values Table, including Standard
the USMLE website or by your registration entity International conversions, is reproduced on pages
on CD. 36B37 of this booklet. This table will be available as an
on-line reference when you take the examination. Please
The information in this booklet, USMLE sample test note that values shown in the actual examination may
materials and software tutorials, and other differ slightly from those printed in this booklet. Other
informational materials are available at the USMLE computer interface features include clickable icons for
website (http://www.usmle.org). Information regarding marking questions to be reviewed, automated review of
any changes in the USMLE program will also be marked and incomplete questions, a clock indicating the
posted at the USMLE website. You must obtain the time remaining, and a help application. This will
most recent information to ensure an accurate provide examinees with a realistic understanding of the
understanding of current USMLE rules. computer interface and timing of the examination.
Preparing for the Test, Applying for the Test,
Scheduling Test Dates, and Testing Examination Format

Step 3 consists of multiple-choice items and computer-


In addition to the information in this booklet, you based case simulations, distributed according to the
should review the sections that appear on pages 12B25 content blueprint. The examination material is prepared
in the Bulletin: Preparing for the Test, Applying for the by examination committees broadly representing the
Test and Scheduling Your Test Date, and Testing. medical profession. The committees comprise
Although the sample test materials in this booklet are recognized experts in their fields, including both
the same as those provided in computer format at the academic and non-academic practitioners, as well as
USMLE website and on CD, you must run the tutorial members of state medical licensing boards.
and sample materials to become familiar with the test Step 3 is a two-day examination. You must complete
software prior to your test date. It is essential that you each day of testing within 8 hours. The first day of
practice with the Primum® Computer-based Case testing includes 336 multiple-choice items divided into
Simulation (CCS) format on the computer prior to blocks of 48 items. There will be 60 minutes of time
taking the examination. Experience shows that those allowed for completion of each block of test items.
who do not practice with the format and mechanics of There is a maximum of 7 hours of testing on the first
managing the patients in Primum CCS are likely to be day. There is also a minimum of 45 minutes of break
at a disadvantage when taking the cases under time and a 15-minute optional tutorial. Note that the
standardized testing conditions. At the time of your test amount of time available for breaks may be increased
appointment, an optional CCS tutorial will be offered, by finishing a block of test items or the optional tutorial
but no practice cases will be available. before the allotted time expires.

2
The second day of testing includes 144 multiple-choice A sample Laboratory Values table of frequently
items, divided into blocks of 36 items. These blocks ordered laboratory tests, including Standard Interna-
will take 45 minutes. Approximately 3 hours are tional conversions, is available in each test item block
allowed for these multiple-choice item blocks. The as a button that retrieves the table for easy viewing.
second day also includes a CCS tutorial for which This table is shown on pages 36–37.
approximately 15 minutes are allowed. This is followed
by approximately 9 case simulations, for which Primum Computer-based Case Simulations. You will
approximately 4 hours are allotted. A minimum of manage one case at a time. Free-text entry of patient
45 minutes is available for break time. There is an orders is the primary means for interacting with the
optional survey at the end of the second day, which can format. Selection of buttons and check boxes is used
be completed if time allows. for advancing the clock, changing the patient’s location,
reviewing previously displayed information, and
Multiple-choice Items. One-best-choice formats are obtaining updates on the patient.
used. Items may stand alone or may be sequenced
together as a case or set of 2 to 3 items. It will be At the beginning of each case, you will see the clinical
useful to study the descriptions on pages 23–24 and to setting, simulated case time, and introductory patient
complete the sample test items provided on the CD and information. Photographs and sounds will not be
in this book starting on page 39. Test items present provided. Normal or reference laboratory values will be
detailed clinical situations, usually from the patient's provided with each report; some tests will be
perspective. The presentation may be supplemented by accompanied by a clinical interpretation. To manage
one or more pictorials. Assessing the patient’s situation patients using the Primum CCS software, it is essential
in the context of his or her environment or family is an that you complete the tutorial and sample cases
important element of many Step 3 questions. provided on the CD.

As is done for the actual examination, the sample test Purpose of the Examination
items are arranged in blocks that are organized by one
The purpose of Step 3 is to determine if a physician
of the three clinical settings described on page 5. The
possesses and can apply the medical knowledge and
amount of time allotted to complete each block of the
understanding of clinical science considered essential
sample test is proportional to the amount of time that
for the unsupervised practice of medicine, with
will be available for each block of questions in the
emphasis on patient management in ambulatory care
actual examination. During the time allotted to
settings. The inclusion of Step 3 in the USMLE
complete the test items in a block, examinees may
sequence of licensing examinations ensures that
answer the items in any order, review responses, and
attention is devoted to the importance of assessing the
change answers. After exiting a block, no further
knowledge and skills of physicians who are assuming
review of items or changing of answers within that
independent responsibility for providing general
block is possible. Policies regarding review of test items
medical care to patients.
may be changed without notice. The most current
policies regarding review are provided on the Internet # Step 3 emphasizes selected physician tasks,
(www.usmle.org). The computer interface includes, namely, evaluating severity of patient problems
among other features, clickable icons for marking and managing therapy. Assessment of clinical
questions to be reviewed, automated review of marked judgment will be prominent.
and incomplete items, and a clock indicating the time
remaining. Practice with the multiple-choice items on # Clinical problems involve mainstream, high-
the CD will provide examinees with a realistic impact diseases. Provision is made for less
understanding of the computer interface and timing of common but important clinical problems
the examination. as well.

# Test items and cases are patient centered, encounter (vignette). Both the multiple-choice
starting with a description of a clinical items and case simulations pose action-related

3
challenges that require clinical # Initial workup encounters are presentations of new,
decisions or judgment. acutely occurring problems among patients seen for the
first time. Tasks emphasized include extensive data
# Emphasis is on ambulatory patient encounters; gathering and initial therapeutic intervention.
however, inpatient encounters of significant
complexity and reflecting contemporary trends # Continuing care encounters are characterized by
also are represented. management of previously diagnosed clinical problems
among patients seen principally in ambulatory settings.
# Provision is made for incorporating applied Evaluating the severity of the patient’s problem(s) and
basic and clinical science concepts, especially prognosis, monitoring therapy, and long-term
as they relate to justification for prognosis or management are emphasized.
management. It is assumed that basic science
and clinical fundamentals have been assessed # Urgent intervention encounters include life and/or
adequately in the prerequisite Step 1 and Step organ-threatening emergencies usually occurring in
2 examinations. emergency department or inpatient settings. Physician
tasks emphasized include rapid assessment of complex
Examination Design presentations and prompt therapeutic decision making.
The principal organizing dimension for Step 3 design is
Much of the test material relates to continuing care
the clinical encounter frame. The concept of frames
encounters. Hence, the bulk of Step 3 is intended to
encompasses several elements that are critical to the
challenge you to consider the severity of illness and to
definition of a patient-physician encounter. These
manage ambulatory patients who have previously
elements include whether the problem or concern is new
diagnosed, frequently occurring chronic illnesses and
or ongoing, the urgency of the need for intervention
behavioral/emotional problems. The Step 3 blueprint is
relative to the underlying problem, the chronology of
shown in Figure 3 on page 8.
events, and the degree of familiarity with the patient or
the patient’s history. In addition, each encounter
The Problem/Disease List on pages 9–18 is derived
between patient and physician occurs in a specifically
from a model of practice for USMLE. The categories
defined location. The primary encounter frames
and content coverage in these materials describing
include: (1) initial workups; (2) continuing care; and
Step 3 are subject to change.
(3) urgent intervention. A more detailed description of
these frames is contained in Figure 1 on page 6.
Clinical Context of Step 3
The second organizing dimension for Step 3 design is
Step 3 is the final examination in the USMLE sequence
the physician task, with six categories: (1) obtaining
leading to a license to practice medicine without
history and performing the physical examination; (2)
supervision. The test items and cases reflect the clinical
using laboratory and diagnostic studies; (3) formulating
situations that a general, as yet undifferentiated,
the most likely diagnosis; (4) evaluating the severity of
physician might encounter within the context of a
the patient’s problems; (5) managing the patient
specific setting. In addition, the items in each test are
(including health maintenance, interventions,
usually arranged by the setting in which the encounter
therapeutics, applying legal and ethical principles); and
first occurs. There are three settings described here. To
(6) applying basic science concepts (mechanisms). See
help orient you, each setting is described at the
Figure 2 on page 7 for a more detailed description.
beginning of the corresponding test block.

The expected outcome of the USMLE process is a


general unrestricted license to practice medicine
without supervision. Although you may already have
begun specialist training, for this examination you are
expected to assume the role of a general, as yet
undifferentiated, physician. You are a member of an
independent group practice affiliated with a number of

4
managed care organizations. Your office has regularly occasion you see patients in the critical care unit.
scheduled hours. You can admit patients to a 400-bed Postoperative patients are usually seen in their rooms
regional hospital, which provides care for both the unless the recovery room is specified. You may also be
urban and the outlying rural communities. The hospital called to see patients in the psychiatric unit. There is a
provides standard diagnostic, radiologic, and short-stay unit where you may see patients undergoing
therapeutic options, including ICUs and cardiothoracic same-day operations or being held for observation.
surgery. There is a labor and delivery suite. A fully Also, you may visit patients in the adjacent nursing
equipped emergency department adjoins the hospital, home/extended care facility and detoxification unit.
and medical evacuation helicopter service is available
for emergency transfer to a regional trauma center. Setting III: Emergency Department
You do not have specialty-oriented hospital privileges,
but you may request any specialty consultation. The Most patients in this setting are new to you, but
laboratory values on pages 36–37 are the normal occasionally you arrange to meet there with a known
ranges for this hospital. patient who has telephoned you. Generally, patients
encountered here are seeking urgent care. Also
Step 3 patients are intended to reflect the diversity of available to you are a full range of social services,
health care populations with respect to age, sex, including rape crisis intervention, family support, and
cultural group, and occupation. The patient population security assistance backed up by local police.
mix is intended to be representative of data collected
from various national databases that study health care
in the United States.

Clinical Settings

The multiple-choice items are organized into blocks


that correspond to the clinical settings in which you will
encounter patients. Each setting is described at the
beginning of its block; these descriptions are shown
here as they would appear during the examination.

Setting I: Office/Health Center

You see patients in two locations: your office suite,


which is adjacent to a hospital, and at a community-
based health center. Your office practice is in a primary
care generalist group. Patients are seen for routine and
urgent care at the office and health center. Most of the
patients you see are from your own practice, although
occasionally you will see a patient cared for by one of
your associates and reference may be made to the
patient's medical records. Known patients may be
managed by telephone, and you may have to respond to
questions about information appearing in the public
media, which will require interpretation of the medical
literature. The laboratory and radiology departments
have a full range of services available.

Setting II: Inpatient Facilities

You have general admitting privileges to the hospital,


including to the children’s and women’s services. On

5
Content Description

The content description that follows is not intended as a study guide, but rather is a model of the range of challenges
that will be met in the actual practice of medicine. Successful completion of at least one year of postgraduate training
in a program accredited by the Accreditation Council for Graduate Medical Education or the American Osteopathic
Association should be helpful preparation for Step 3.

The following pages provide a description of the content of the Step 3 examination. Figures 1 and 2 are complete
descriptions of the two major Step 3 design dimensions, clinical encounter frames and physician tasks.

Figure 1: Step 3 Clinical Encounter Frames

INITIAL WORK-UP CONTINUING CARE URGENT INTERVENTION


1 2 3

Patient encounters characterized by Patient encounters characterized by Patient encounters characterized


initial assessment and management continuing management of by prompt assessment and
of clinical problems among patients previously diagnosed clinical management of life-threatening
seen principally in ambulatory problems among patients known to and organ-threatening
settings for the first time. These the physician and seen principally emergencies, usually occurring
encounters may also include new in ambulatory settings. in emergency department
problems arising in patients for Encounters focused on health settings.
whom a history is available. maintenance are located in this
frame. Occasionally, these encounters
may occur in the context of a
Also included are patient encounters hospitalized patient.
characterized by acute
exacerbations or complications,
principally of chronic, progressive
conditions among patients known to
the physician. These encounters
may occur in inpatient settings.

Clinical problems include ill-


defined signs and symptoms; Clinical problems include Clinical problems include severe
behavioral-emotional; acute frequently-occurring chronic life-threatening and organ-
limited; initial manifestation and diseases and behavioral-emotional threatening conditions and
presentation of chronic illness. problems. Periodic health exacerbations of chronic illness.
evaluations of established patients
are included here.
Physician tasks emphasized include
data gathering and initial clinical Physician tasks emphasized include Physician tasks emphasized
intervention. Assessment of recognition of new problems in an include rapid assessment of
patients may lead to urgent existing condition, assessment of complex presentations,
intervention. severity, establishing prognosis, assessment of patients'
monitoring therapy, and long-term deteriorating condition, and
management. prompt decision making.

6
Figure 2: Step 3 Physician Tasks

Obtaining history ! Objectives focus on interpreting the patient’s history, knowing pertinent
and performing factors in the patient’s history, and interpreting the history in terms of risk
physical factors for the patient.
examination ! Objectives focus on the physical examination, such as recognizing and
interpreting pertinent physical findings and knowing required techniques in
the physical examination.
Using laboratory ! Objectives focus on selecting the appropriate routine, initial, invasive,
and diagnostic special, or follow-up studies; interpreting the results of laboratory or
studies diagnostic tests; knowing the value of and indications for screening tests;
and predicting the most likely test result.
Formulating the ! Objectives focus on selecting the most likely diagnosis in light of history,
most likely physical, or diagnostic test findings. Includes interpreting pictorial material
diagnosis and establishing a diagnosis.
Evaluating severity ! Objectives focus on interpreting the vignette, evaluating the severity of the
of patient’s patient’s condition, and making judgment on the current status or prognosis
problems of the patient as to the need for further action.
Managing the ! Health maintenance objectives focus on identifying risk factors, knowing
patient incidence within patient groups at risk, knowing preliminary steps to ensure
effectiveness of intended therapy, and selecting appropriate preventive
therapeutic agents or techniques.
! Clinical intervention objectives focus on knowing priorities in emergency
management, knowing present and long-term management of selected
conditions, and knowing appropriate surgical treatment, including pre- and
post-surgical events. They also include knowing pre- and post-procedural
management and the appropriate follow-up schedule or monitoring
approach.
! Clinical therapeutics objectives focus on selecting the appropriate
pharmacotherapy, recognizing actions of drugs as applied to patient
management, and knowing the importance of educating patients about
effects of drugs and drug-drug interactions.
! Legal/ethical and health care systems objectives focus on issues such as
patient autonomy, physician/patient relationships, use of unorthodox or
experimental therapies, end-of-life considerations, treatment of minors, and
physician error versus negligence.
Applying ! Objectives focus on identifying the underlying processes or pathways
scientific responsible for a given condition, recognizing associated disease conditions
concepts and complications, and recognizing and evaluating clinical findings or
diagnostic studies to identify the underlying factors (eg, anatomic structure).
! Objectives focus on interpreting results of experimental or biometric data,
including knowing design features of clinical studies, understanding issues
regarding validity of research protocols, knowing sensitivity and specificity
of selected tests, and recognizing potential bias in clinical studies.

7
Figure 3 shows how frames and tasks intersect to create the Step 3 blueprint that specifies the broad content allocations for
constructing Step 3. Estimates of approximate percentages are provided for the marginal totals.

Figure 3: Step 3 Blueprint

CLINICAL ENCOUNTER FRAMES

PHYSICIAN TASKS Initial Work-up Continued Care Urgent Intervention

1 2 3 TOTAL

History & Physical 8–12%

Diagnostic Studies 8–12%

Diagnosis 8–12%

Prognosis 8–12%

Managing Patients

Health Maintenance 5–9%

Clinical Intervention 18–22%

Clinical Therapeutics 12–16%

Legal & Ethical Issues 4–8%

Applying Basic Concepts 8–12%

TOTAL 20–30% 50–60% 15–25% 100%

8
Problem/Disease List

The Step 3 Problem/Disease List provided here is organized at the system level according to the International
Classification of Diseases, 9th Revision, Clinical Modification (ICD-9-CM). It is subdivided according to
presentation categories and includes disorders, health problems, and issues relating to health maintenance. This list
has been prepared from empirical data drawn from several sources, including, for example, the National
Ambulatory Medical Care Survey and the National Hospital Discharge Survey. The aggregated frequency listings
have been subjected to extensive review. Although the Problem/Disease List is not comprehensive, it does serve as a
primary reference for identifying the clinical problems the generalist physician should be prepared to encounter in
one or more of the frames cited.

DISEASES/DISORDERS OF THE CENTRAL NERVOUS SYSTEM

Infectious Diseases of CNS Vascular Diseases of CNS


Tetanus Cerebral hemorrhage
Poliomyelitis and other nonarthropod-borne Intracranial hemorrhage
viral diseases Transient cerebral ischemia
Creutzfeldt-Jakob diseases Cerebrovascular disease
Rabies Occlusion/stenosis of precerebral/cerebral
Meningitis (Bacterial, Viral, Other) arteries
Encephalitis Neoplasms
Brain abscess Malignant intracranial neoplasm
Degenerative/Hereditary Diseases of CNS Malignant neoplasm of nervous system
Alzheimer disease Ill-defined, Symptoms referable to the Nervous
Other cerebral degenerations System
Parkinson disease Migraine
Movement disorders, excluding Parkinson's Headache
Multiple sclerosis Alteration of consciousness (delirium)
Amyotrophic lateral sclerosis Hallucinations
Other diseases of spinal cord Syncope and collapse
Reflex sympathetic dystrophy Dizziness/giddiness
Other Diseases of CNS Abnormal involuntary movement
Mental retardation Ataxia
Quadriplegia and other paralytic syndromes Speech disturbance
Seizure disorders Other general symptoms
Cerebral palsy Sleep disorders
Unspecified diseases of CNS Coma
Diseases of Peripheral Nerves Coma
Facial nerve disorders Brain death
Neuropathies
Mononeuritis
Myasthenia gravis
Myopathy, unspecified
Neuralgia/Neuritis

9
DISEASES/DISORDERS OF THE EYE
Eye
Ophthalmic manifestations of diabetes Diplopia
Degeneration of macula Visual field defects
Retinal detachments, defects, and disorders Disorders of conjunctiva
Chorioretinitis Disorders of eyelids/lacrimal system
Glaucoma Orbital cellulitis
Cataract Diseases of optic nerve/visual pathways
Myopia Strabismus
Visual disturbances (incl. sudden visual loss, Other disorders of the eyes (incl. nystagmus)
psychophysical visual disturbance) Corneal abrasion

DISEASES/DISORDERS OF THE EAR/NOSE/MOUTH/THROAT


Ear Sinuses
Diseases of ear and mastoid process Acute sinusitis
Otitis media Chronic sinusitis
Disorders of tympanic membrane Mouth
Vertiginous syndromes and labyrinthine Teething syndrome
disorders Dental caries
Tinnitus Stomatitis
Otalgia Disorders of teeth/jaw
Hearing loss
Neoplasms of Head and Neck
Nose/Throat Malignant neoplasm of larynx
Acute respiratory infections Malignant neoplasm of lip, oral cavity, and
Chronic respiratory infections pharynx
Peritonsillar abscess
Allergic rhinitis
Diseases of nasal cavity, sinuses
Epistaxis
Other diseases of upper respiratory tract

DISEASES/DISORDERS OF THE RESPIRATORY SYSTEM


Acute Respiratory Infections Other Diseases of the Respiratory System
Croup Pulmonary tuberculosis
Acute bronchitis Sarcoidosis
Acute bronchiolitis Cystic fibrosis
Bronchopneumonia Pulmonary embolism and infarction
Pneumonia/Influenza Pulmonary hypertension
Viral pneumonia Pleurisy without effusion
Bacterial pneumonia Spontaneous tension pneumothorax
Influenza with other resp. manifestation Postinflammatory pulmonary fibrosis
Chronic Obstructive Pulmonary Disease Other diseases of trachea and bronchus, NEC
Bronchitis Pertussis
Obstructive chronic bronchitis Mediastinitis
Emphysema Wegener granulomatosis
Asthma
Chronic airway obstruction

10
Lung Disease Due to External Agent Ill-Defined Symptoms Referable to Respiratory
Pneumoconiosis System
Neoplasms Respiratory abnormalities
Malignant neoplasm of trachea, bronchus, and
lung
Malignant neoplasm of pleura
Secondary malignant neoplasm of respiratory
system

DISEASES/DISORDERS OF THE CIRCULATORY SYSTEM


Hypertensive Disease Peripheral Vascular Disease
Essential hypertension Aortic aneurysm
Hypertensive heart and renal diseases Other peripheral vascular diseases
Secondary hypertension Arterial embolism/thrombosis
Ischemic Heart Disease Other diseases of arteries and arterioles
Acute myocardial infarction Other and unspecified capillary diseases
Other acute/subacute forms of ischemic heart Phlebitis/Thrombophlebitis
disease Venous embolism/thrombosis
Angina pectoris Varicose veins
Other forms of chronic ischemic heart disease Other specified disorders of circulatory system
Other Forms of Heart Disease Atherosclerosis
Pericarditis Atherosclerosis
Cardiomyopathy Valvular Heart Disease
Congestive heart failure Rheumatic heart disease
Myocarditis Diseases of mitral, aortic, and tricuspid valves
Arrhythmias/Conduction Disorders Other diseases of endocardium
Conduction disorders Endocarditis
Cardiac dysrhythmias Congenital anomalies of the heart
Ill-defined Symptoms Referable to Circulatory
System
Symptoms involving the CV system
Elevated blood pressure reading without
hypertension diagnosis

DISEASES/DISORDERS OF THE DIGESTIVE SYSTEM


Esophagus Small Intestine/Colon
Malignant neoplasm of esophagus Malignant neoplasm of colon
Esophageal varices Benign neoplasm of duodenum, jejunum, and
Diseases of esophagus ileum
Stomach Appendicitis
Malignant neoplasm of stomach Other noninfectious gastroenteritis
Benign neoplasm of stomach Intestinal obstruction
Gastric ulcer problems Diverticula of small intestine
Duodenal ulcer problems Diverticula of colon
Peptic ulcer problems Irritable colon/Irritable bowel syndrome
Gastritis/Duodenitis Peritonitis
Other disorders of intestines

Rectum Pancreas

11
Malignant neoplasm of rectosigmoid junction Malignant neoplasm of pancreas
Malignant neoplasm of rectum Benign neoplasm of pancreas
Malignant neoplasm of anus Diseases of pancreas
Benign neoplasm of rectum Hernia
Hemorrhoids Hernia
Anal fissure Ventral hernia
Anal fistula Diaphragmatic hernia
Abscess of anal and rectal regions Ill-defined Symptoms Referable to Digestive
Other disorders of rectum and anus System
Gallbladder Dyspepsia
Calculus of gallbladder Vomiting
Cholecystitis Constipation
Cholangitis Diarrhea
Liver Hemorrhage of the GI tract
Malignant neoplasm of liver Symptoms involving the digestive system
Cirrhosis of liver Other symptoms involving abdomen and pelvis
Obstruction of common bile duct Abnormal stool contents

BEHAVIORAL/EMOTIONAL DISORDERS

Schizophrenia & Psychotic Disorders Disorders Originating in Childhood


Schizophrenia Psychoses originating in childhood
Paranoid states (including delusional disorders) Conduct disturbance
Psychotic disorder Attention deficit/hyperactivity disorder
Anxiety Disorders Autistic disorder
Panic attacks Developmental speech or language disorder
Generalized anxiety disorder Cognitive Disorders
Phobic disorders Vascular dementia
Obsessive-compulsive disorder Postconcussion syndrome
Acute stress disorder Substance Abuse Disorders
Adjustment disorders Alcohol-related disorders
Bereavement Chemical/Drug dependence/abuse
Mood Disorders Psychosocial Problems
Major depressive disorders Abuse of other person
Other depressive disorders Relational problems
Bipolar disorders Sexual dysfunction
Manic disorders Personality Disorders
Somatoform Disorder Personality disorders
Somatization disorder Movement Disorders
Malingering Tardive dyskinesia
Conversion disorder Akathisia, drug-induced
Factitious disorder
Hypochondriasis
Eating Disorders & Other Behavioral
Disorders
Pica
Anorexia
Bulimia
Disorders of impulse control

DISEASES/DISORDERS OF THE MUSCULOSKELETAL SYSTEM

12
Arthropathies and Related Disorders Malignancy
Diffuse diseases of connective tissue Secondary malignant neoplasm of
Rheumatoid arthritis bone/marrow
Internal derangement of knee Pathologic fracture
Effusion of joint Fractures/Dislocations
Rheumatism Excluding the Back Closed fracture of mandible/facial bones
Affections of shoulder region Fracture of vertebral column
Enthesopathy Fracture of ribs
Disorders of synovium, tendon, and bursa Closed fracture, upper extremity
Disorders of muscle, ligament, and fascia Fracture of femur
Rheumatism Various closed fractures of lower extremity
Myalgia/Myositis Dislocations/Separations
Other diseases of soft tissues Sprains/Strains
Osteopathies, Chondropathies, and Acquired Temporomandibular joint disorders
Musculoskeletal Deformities Rotator cuff syndrome
Disorders of bone and cartilage Enthesopathy of elbow, hip, and ankle
Deformities and anomalies of limbs/head Various sprains and strains
Congenital musculoskeletal deformities Contusions
Disorders of Back/Spine/Cervical Region Various contusions
Spinal enthesopathy Arthropathies and Related Disorders
Spondylosis Infective arthritis
Intravertebral disc disorders Osteoarthritis
Disorders of cervical region Monoarthritis
Spinal stenosis Arthropathy
Disorders of back Polyarthritis
Kyphoscoliosis and scoliosis Ill-defined Symptoms Referable to the
Lumbosacral sprain Musculoskeletal System
Sprain of sacroiliac region/back Pain in joint
Contusion of back Stiffness of joint
Pain in thoracic spine
Pain in limb
Other MSK symptoms in limbs

DISEASES/DISORDERS OF THE SKIN/SUBCUTANEOUS TISSUE


Skin Eruptions Lumps/Tumors of the Skin
Infections and parasitic diseases Viral warts
Other inflammation of skin and subcutaneous Malignant neoplasms of skin
tissue Benign neoplasm of skin
Other disorders of skin and subcutaneous Keratoderma, acquired
tissue Sebaceous cyst
Nails/Hair Neurofibromatosis
Symptoms of the nail Lipoma
Diseases of hair and follicle Ill-defined Skin and Integumentary
Symptoms involving skin and integumentary
tissue
Localized superficial swelling, mass, or lump
Hyperhidrosis
Keloid

DISEASES/DISORDERS OF ENDOCRINE/NUTRITION/METABOLISM

13
Diseases of the Thyroid Gland Nutritional Deficiencies
Malignant neoplasm of thyroid gland Malnutrition
Disorders of the thyroid gland Vitamin and mineral deficiencies
Diabetes Mellitus Other Metabolic/Immunity Disorders
Diabetes mellitus Phenylketonuria (PKU)
Gangrene Disorders of lipoid metabolism
Disorders of Endocrine Glands Gout
Hypoglycemia Disorders of mineral metabolism
Hyperglycemia Electrolyte and fluid disorders
Disorders of the parathyroid gland Immunity deficiency
Endocrine disorders
Disorders of adrenal gland

DISEASES/DISORDERS OF THE KIDNEYS AND URINARY TRACT

Lower Upper
Malignant neoplasm of prostate Malignant neoplasm of kidney
Malignant neoplasm of bladder Nephrotic syndrome/nephritis
Neurogenic bladder Chronic renal failure/insufficiency
Urethritis Acute renal failure
Other disorders of urinary tract Cystic kidney disease
Enuresis Urinary calculus
Renal colic
Ill-defined Symptoms Referable to
Genitourinary System
Stress Incontinence, female
Symptoms referable to genitourinary system

DISEASES/DISORDERS OF THE MALE REPRODUCTIVE SYSTEM

Male Reproductive System


Malignant neoplasm of male breast Symptoms referable to testes
Malignant neoplasm of prostate Orchitis/Epididymitis
Malignant neoplasm of male reproductive Male infertility
system Other disorders of male genital organs
Disorders of prostate

DISEASES/DISORDERS OF THE FEMALE REPRODUCTIVE SYSTEM


Uterus Tube and Adnexa
Malignant neoplasm of uterus Malignant neoplasm of ovary
Leiomyoma of uterus Ovarian cyst
Other disorders of uterus Other ovarian failure
Endometriosis of uterus Benign neoplasm of ovary
Uterine prolapse Acute parametritis and pelvic cellulitis
Abnormal uterine bleeding Noninflammatory disorders of ovary, fallopian
tube, and broad ligament
Inflammatory disorders of ovary

Cervix Menstrual Disorders


Malignant neoplasm of cervix Dysmenorrhea

14
Cervicitis/Endocervicitis Premenstrual tension
Dysplasia of cervix uteri Disorders of menstruation
Abnormal Pap smear Menopause
Vagina/Vulva Menopausal symptoms
Malignant neoplasm of vagina Breast
Malignant neoplasm of vulva Malignant neoplasm of breast
Vaginitis/Vulvovaginitis Benign neoplasm of breast
Prolapse of vaginal walls Breast related problems
Imperforate hymen Signs and symptoms in breast
Noninflammatory disorders of vagina Female Infertility
Disorders of pelvic organs
Female infertility

PREGNANCY/CHILDBIRTH

Pregnancy, Labor, and Delivery–Complicated Pregnancy, Labor, and Delivery–Uncomplicated


Adolescent pregnancy Delivery and labor with minor or no
Pregnancy loss complications
Pregnancy complication Supervision of normal pregnancy
Eclampsia Single liveborn before admission to hospital
Complications of labor, delivery
Cesarean delivery
Chromosomal anomalies
Multiple fetuses

NEONATE/CHILD

Conditions Originating in the Perinatal


Period Hypocalcemia
Congenital anomalies Conditions specific to the perinatal period
Fetal growth retardation Hemolytic disease due to Rh
Post-term infant isoimmunization
Birth trauma Perinatal jaundice
Respiratory problems after birth Feeding problems in newborn

DISEASES/DISORDERS OF BLOOD/BLOOD-FORMING ORGANS


Malignant Neoplasias Other Conditions Referable to
Hodgkin disease Blood/Blood-Forming Organs
Lymphomas Septicemia
Multiple myeloma and Immunoproliferative Agranulocytosis
neoplasms Polycythemia
Leukemias Diseases of the spleen
Anemias Viremia
Anemias Transfusion reaction
Bleeding Disorders
Coagulation disorders
Thrombocytopenia

INFECTIOUS/PARASITIC DISEASES

15
Intestinal Infectious Diseases Other Diseases Due to Viruses and Chlamydiae
Intestinal Infectious Diseases Hepatitis
Other Bacterial Diseases Mumps
Pertussis Specific diseases due to Coxsackie virus
Streptococcal sore throat Infectious mononucleosis
Streptococcus infection Trachoma
Staphylococcus infection Diseases of conjunctiva
Pneumococcus infection Molluscum contagiosum
Escherichia coli infection Cat-scratch disease
Haemophilus influenzae infection Foot and mouth disease
Proteus infection Cytomegalic inclusion disease
Pseudomonas infection Other diseases due to viruses and chlamydiae
Other bacterial diseases Adenovirus
HIV Infection ECHO virus
AIDS, HIV infection, AIDS-Related Complex Coxsackievirus
(ARC) Rhinovirus
Pneumocystosis (pneumocystis) Human papillomavirus (HPV)
Kaposi sarcoma Retrovirus
Viral Diseases Accompanied by Exanthem Respiratory syncytial virus
Chickenpox Other specified viral and chlamydial infections
Herpes zoster without complications Toxoplasmosis
Herpes simplex Arthropod-Borne Diseases
Herpes zoster with herpes simplex West Nile virus/Fever
Herpes simplex fever blisters Rocky mountain spotted fever
Measles Rickettsiosis
Rubella and roseola Malaria
Erythema infectiosum (Fifth disease) Lyme disease
Kawasaki disease

Sexually Transmitted Diseases Infectious Diseases Complicating


Syphilis Pregnancy/Childbirth

16
Gonococcal infections Infections of genitourinary tract during
Other sexually transmitted disease pregnancy
Mycoses Other infections complicating
Dermatophytosis pregnancy/childbirth
Candidiasis Infections of Skin and Subcutaneous Tissue
Mycoses Cellulitis and abscess
Infectious Diseases of Urinary System Impetigo
Infections of kidney Other local skin infection
Cystitis Infections Involving Bone
Urinary tract infection Acute and chronic osteomyelitis

INJURIES/WOUNDS/TOXIC EFFECTS/BURNS

Eye Injury Trauma


Foreign body in eye Rape/Crisis adjustment
Eye burn Concussion
Ocular injuries Cranial injuries
Wounds Chest trauma
Various open wounds (includes animal bites) Internal injuries, abdomen and pelvis
Foreign Body Other trauma
Foreign body in ear Frostbite
Foreign body in nose Heat stroke
Foreign body in pharynx & larynx Complications of Surgery and Medical Care
Foreign body in trachea CNS complications
Swallowed foreign body Respiratory complications
Various Toxics Digestive system complications
Food poisoning Hemorrhage complicating a procedure
Poisonings by drugs and medicinal substances Postoperative infection
Toxic effects of nonmedicinal substances Other infection following medical care
Adverse effects of drug, medicinal, and Other complications of medical procedures
biological substance Other complications of internal device
Burns
Burns

HEALTH MAINTENANCE

General Checks Targeted Checks, Gynecology


Community-related prevention Contraceptive management
Routine child/infant health check Genetic counseling
Specific procedures and other follow-up care Antenatal screening
Counseling Gynecologic examination
Follow-up examinations Post-partum follow up
General medical examination Cancer screening
Laboratory examination Cardiovascular risk
Cardiovascular risk screening and counseling
Exercise

Vaccinations/Other Chemotherapy Abuse of Drugs


Vaccinations Alcohol abuse

17
Desensitization to allergens Tobacco use disorder
Prophylactic chemotherapy Cannabis abuse
Chemotherapy Hallucinogen abuse
Skin sensitization tests Sedative, hypnotic, anxiolytic abuse
Opioid abuse
Cocaine abuse
Amphetamine abuse
Antidepressant type abuse
Other mixed, unspecified drug abuse

ILL-DEFINED SYMPTOM COMPLEXES

Symptoms Referable to Nutrition Ill-Defined Causes of Morbidity


Abnormal weight gain Malignant neoplasm, unspecified site
Abnormal weight loss Secondary malignant neoplasm, other sites
Loss of appetite Palliative care
Ill-defined Presentations
Pyrexia (fever) of unknown origin
Malaise and fatigue
Abnormality of gait
Feeding difficulties
Lack of normal physiological development
Throat pain
Epistaxis
Septic shock
Chest pain
Asphyxia
Nervousness
Allergy
Chronic pain

18
Evaluative Objectives
A complete listing of Step 3 Evaluative Objectives follows. These objectives are categorized according to the physician
tasks and they serve to guide writing and classification of test items. They can be read as more detailed descriptions
of the kinds of issues that will be posed to physicians taking Step 3.

Step 3 Evaluative Objectives

Objectives Related to Obtaining History and Objectives Related to Using Laboratory


Performing Physical Examination and Diagnostic Studies

# Knows signs/symptoms of selected disorders. # Knows when NO diagnostic or laboratory


studies are indicated.
# Interprets elicited history.
# Selects appropriate routine or initial laboratory
# Knows individual’s risk factors for
or diagnostic studies.
development of condition leading to encounter.
# Interprets the clinical impact of laboratory or
# Knows commonly associated pertinent
diagnostic test findings.
history.
# Knows appropriate invasive, special,
# Associates current complaints with presented
non-routine, or follow-up studies.
history and identifies pertinent factor.
# Knows appropriate common screening tests for
# Knows appropriate directed physical
conditions related to an encounter or complaint.
examination or required technique.
# Identifies study/procedure most likely to
# Recognizes pertinent physical findings.
establish the diagnosis or distinguish one
# Interprets particular physical finding in order diagnosis from another.
to differentiate one disorder from another.
# Predicts the most likely laboratory or diagnostic
# Interprets mental status or identifies neurologic test result.
exam findings.

19
Objectives Related to Formulating the Objectives Related to Managing the Patient
Most Likely Diagnosis Management of Health Maintenance
and Disease Prevention
# Selects the most likely diagnosis or evaluates # Recognizes physician’s best choice of words in
differential in light of history and/or physical eliciting history or further description from the
and/or diagnostic test findings. patient; knows statements that facilitate communi-
# Knows the most likely cause or preliminary cation with the patient.
diagnosis associated with the presented signs # Knows risk factors for conditions amenable to
and symptoms or physical findings. prevention or detection in an asymptomatic pa-
tient.
# Knows pertinent incidence statistics and identifies
Objectives Related to Evaluating the Severity
patient groups at risk; knows incidence of
of Patient’s Problems (Prognosis)
symptomless/dangerous disorders among various
# Evaluates severity of patient condition and groups.
identifies indications for consultation or
diagnostic assessment. # Knows appropriate directed physical examination
in screening an asymptomatic patient during a
# Evaluates appropriateness of available data or well-care visit.
therapies.
# Knows common screening tests for conditions
# Assesses severity of patient condition and amenable to prevention or detection in an
makes judgment as to current status, prognosis, asymptomatic patient or population.
or need for further action.
# Knows range and timing of diagnostic study in
# Recognizes factors in the history (given monitoring for prevention.
symptoms) that affect patient prognosis.
# Selects appropriate preventive, therapeutic
# Recognizes iatrogenic complications, not agent/technique.
drug-related.
# Knows clinically relevant implications of
specifically referenced treatment.
# Interprets laboratory or diagnostic results and
identifies current status of patient.

20
Clinical Intervention
# Knows appropriate follow-up schedule or
# Evaluates severity of patient condition in terms monitoring approach regarding therapeutic
of need for referral for surgical regimen.
treatments/procedures versus other nonsurgical # Assesses patient compliance with treatment
options. regimen by recognizing techniques to increase
# Knows appropriate next step in patient care. compliance or understanding of the disease state
and how therapy may affect compliance with
# Knows priority in management, specifically in instructions.
emergency or acute cases.
# Applies knowledge of drugs to the safe and
# Knows immediate management or medical effective selection and administration of drugs.
intervention in emergency situations.
# Recognizes factors that alter drug requirements
# Knows appropriate present management of for a patient.
selected conditions.
# Recognizes signs and symptoms of drug
# Knows appropriate long-term treatment or interactions resulting from polypharmacy in the
management goals. therapeutic regimen.
# Knows appropriate surgical management # Knows adverse effects of various drugs.
among surgical options.
# Knows actions in response to acute, specific drug
# Knows postsurgical or postprocedural toxicity.
management.
# Knows contraindications of various drugs.
# Knows indications for hospital admission or
other appropriate setting. # Recognizes new signs and symptoms in a patient
with an established medication regimen within the
# Knows appropriate follow-up schedule or context of continuing care.
monitoring approach regarding the general
management plan. # Modifies therapeutic regimen within the context
of continuing care.
# Knows appropriate discharge planning (eg,
focusing priorities on maximizing benefits of # Knows importance of educating patient or family
hospitalization). specifically about medication regimen.

# Knows components of rehabilitation program. # Applies results of experimental data or biometric


studies to the treatment of a patient.
# Knows various therapeutic modalities for
selected behavioral/affective disorders.
# Knows appropriate counseling of patient or
family regarding current and future problems,
including risk factors related to present
encounter.
# Knows importance of educating patient or
family regarding self-care.
# Knows relevant roles of allied health personnel.

# Knows issues surrounding management of


chronic pain, especially in terminally ill
patients.
Clinical Therapeutics
# Selects most appropriate pharmacotherapy.

21
Legal/Ethical and Health Care Systems Objectives Related to Applying Scientific
# Knows informed consent. Concepts
# Identifies the cause/causal agent or predisposing
# Recognizes patient’s right to refuse treatment or
factor(s); or, given an effect, what is the cause.
testing (patient autonomy).
# Predicts the effect; given a particular cause,
# Knows guidelines for treatment of minors with identifies what effects may be expected. Given an
or without notification of parent. anatomic, pathologic, or physiologic
# Knows guidelines for confidentiality of medical abnormality, predicts the effect on function,
records. including compensatory effects.
# Recognizes opportunities for cost containment. # Recognizes associated disease conditions, in-
cluding complications, or indicators for potential
# Knows guidelines for physician/patient
disease complications, of a given disease
relationship.
(excludes iatrogenic complications).
# Assesses quality of life decisions.
# Identifies the underlying processes/pathways that
# Recognizes priorities in decisions to allocate account for, or contribute to, the expression or
transplant organs and/or other scarce resolution of a given condition.
resources. # Recognizes characteristics of disease relating to
# Knows use of unorthodox/experimental or natural history or course of disease, including
folk/alternative therapies. progression, severity, duration, and transmission
of disease.
# Recognizes physician error vs. negligence.
# Evaluates given clinical or physical findings to
# Knows Good Samaritan laws.
identify the underlying anatomic structure or
# Knows guidelines for reporting findings to physical location.
proper authorities. # Interprets laboratory or diagnostic studies as to
# Recognizes need for third-party permission for the underlying pathophysiology; predicts
treatment in medical emergencies. expected diagnostic test findings for a given
disease.
# Assesses degree of disclosure to terminally ill
patients. # Recognizes appropriate methods and techniques
related to procedures, artifacts of
# Knows definitions of competence and sanity.
instrumentation, technical errors/problems
# Knows guidelines for commitment. contributing to misinformation.
# Recognizes impaired physician. # Interprets results of experimental data or
biometric studies.
# Knows appropriate prescriptive practices.
# Knows design features of clinical studies.
# Knows definition of and legal issues regarding
brain death. # Knows issues regarding validity of research
protocols.
# Knows appropriate use and procedures
regarding hospice care. # Recognizes potential bias in clinical studies
including the extent to which bias accounts for
# Knows appropriate use and procedures study results.
regarding telemedicine.
# Interprets results of clinical studies.
# Distinguishes clinical importance from statistical
significance.
# Knows sensitivity and specificity of selected test.

STEP 3 Test Question Formats Strategies for Answering One Best Answer Questions
(Single Items, Multiple Item Sets, and Cases)

22
# Read the patient description and question carefully.
It is important to understand what is being asked. Example Questions 2 to 4
# Try to generate an answer and then look for it in the A 38-year-old white woman, who is a part-time teacher
option list.
and the mother of three children, comes to the office for
# Alternatively, read each option carefully, evaluation of hypertension. You have been her physician
eliminating those that are clearly incorrect. since the birth of her first child 8 years ago. One week
# Of the remaining options, select the one that is most ago, an elevated blood pressure was detected during a
correct. regularly scheduled examination for entrance into
graduate school. Vital signs on examination today are
# If unsure about an answer, it is better to guess since
unanswered questions are automatically counted as temperature 37.0EC (98.6EF), pulse 100/min,
wrong answers. respirations 22/min, and blood pressure 164/100 mm Hg
(right arm, supine).
Single Items 2. The physical examination is most likely to show
which of the following?
This is the traditional, most frequently used multiple-
choice format. These items usually include a patient
A. An abdominal bruit
vignette followed by four or five response options. The
B. Cardiac enlargement
response options for all questions are lettered (eg, A, B,
C. Decreased femoral pulses
C, D, E). You are required to select the best answer to
D. Thyroid enlargement
the question. Other options may be partially correct, but
E. Normal retinas
there is only ONE BEST answer.
(Answer E)
Example Question 1
3. The most appropriate next step is to order which of
A 45-year-old African-American man comes to the
the following?
office for the first time because he says, "I had blood in
my urine when I went to the bathroom this morning." He
A. Complete blood count
reports no other symptoms. On physical examination his
B. Determination of serum electrolyte and creatinine
kidneys are palpable bilaterally and he has mild
concentrations
hypertension. Specific additional history should be
C. Determination of serum glucose concentration
obtained regarding which of the following?
D. Determination of serum thyroxine concentration
E. Urine culture
A. Chronic use of analgesics
B. Cigarette smoking
(Answer B)
C. A family history of renal disease
D. Occupational exposure to carbon tetrachloride
E. Recent sore throats

(Answer C)

Multiple Item Sets

A single patient-centered vignette may be associated


with two or three consecutive questions about the
information presented. Each question is linked to the
initial patient vignette, but is testing a different point.
Questions are designed to be answered independently of
each other. You are required to select the one best
answer for each question. Other options may be partially
correct, but there is only ONE BEST answer.

23
4. To assess this patient’s risk factors for 5. Which of the following findings on physical
atherogenesis, the most appropriate test is examination would be most consistent with
determination of which of the following? costochondritis as the cause of his chest pain?

A. Plasma renin activity A. Crepitance over the second and third ribs
B. Serum cholesterol concentration anteriorly
C. Serum triglycerides concentration B. Deep tenderness to hand pressure on the sternum
D. Urinary aldosterone excretion C. Localized point tenderness in the parasternal area
E. Urinary metanephrine excretion D. Pain on deep inspiration
E. Normal physical examination
(Answer B) (Answer C)
End of Set
6. In light of the patient’s original denial of drug use,
Cases which of the following is the most appropriate next
step to confirm a diagnosis of cocaine use?
A single-patient or family-centered vignette may ask as
few as two and as many as four questions, each related A. Ask the laboratory if serum is available for
to the initial opening vignette. Information is added as toxicologic screening on a previous blood sample
the case unfolds. It is extremely important to answer B. Call his family to obtain corroborative history
the questions in the order presented. Time often C. Obtain a plasma catecholamine concentration
passes within a case and your orientation to a question D. Obtain a urine sample for routine analysis but
early in a case may be altered by the additional also request toxicologic screening
information presented later in the case. If you do skip E. Present your findings to the patient and confront
questions, be sure to answer earlier questions with only him with the suspected diagnosis
the information presented to that point in the case.
(Answer E)
Each question is intended to be answered independently.
You are required to select the ONE BEST answer to Cocaine use is confirmed. The patient admits a possible
each question. temporal relationship between his cocaine use and his
chest pain and expresses concern about long-term health
risks.
Example Questions 5 to 7
7. The patient should be counseled regarding which of
A 24-year-old man comes to the office because of the following?
intermittent chest pain that began a few weeks ago. You
have been his physician for the past 2 years and he has A. Cocaine-induced myocardial ischemia can be
been in otherwise good health. He says he is not having treated with blocking agents
pain currently. A review of his medical record shows B. Death can occur from cocaine-induced
that his serum cholesterol concentration was normal at a myocardial infarction or arrhythmia
pre-employment physical examination 1 year ago. You C. The presence of neuropsychiatric sequelae from
have not seen him since that visit and he says he has had drug use indicates those at risk for sudden death
no other complaints or problems in the interim. He associated with cocaine use
reminds you that he smokes 1 pack of cigarettes per day. D. Q wave myocardial infarction occurs only with
When you question him further, he says that he does not smoked "crack" or intravenous cocaine use
use any alcohol or illicit drugs. Although the details are E. Underlying coronary artery disease is the
vague, he describes the chest pain as a substernal principal risk for sudden death associated with
tightness that is definitely not related to exertion. cocaine use

(Answer B)
End of Case

24
Primum® Computer-based Case Simulations (CCS) Overview

Introduction requests for interval history and physical examination


This overview, in combination with frequently asked findings, tests, therapies, and procedures. Requests for
questions (FAQs), software instructions, and practice interval history and physical examination automatically
cases is intended to prepare you for an examination that advance the clock in simulated time. To see results of
uses Primum Computer-based Case Simulations (CCS) tests and procedures, and to observe effects of treatment,
software. You will use the Primum program to manage you must advance the clock.
one patient at a time. Each case will be presented in a
consistent format and appearance; the patient Physical examination should be requested if and when
management options will be the same in all cases. you would do the same with a real patient. You can
select a complete physical examination or parts of a
It is highly recommended that you review all Primum physical examination. You can write orders before
orientation materials and manage all practice cases examining a patient; if physical examination reveals
before taking the examination. Experience has shown findings that you believe render the orders inappropriate,
that those who do not practice with the format and and the orders have not yet been processed, you can
mechanics of managing the patients in Primum CCS are cancel those orders. At subsequent intervals of your
likely to be at a disadvantage when taking the cases choosing, you can also request interval histories which
under standardized test conditions. During the are analogous to asking the patient "How are you?"
examination, you will be allotted a maximum of 25
minutes of real time for each case, so familiarity with You will initiate patient care and management actions by
Primum software is essential. Once you are familiar typing on the order sheet. The order sheet enables you to
with the Primum software, you will be able to focus request tests, therapies, procedures, consultations, and
your attention on patient management. nursing orders representing a range of diagnostic and
therapeutic management options. It is also your means
Description of Primum Computer-based Case
of giving advice or counseling a patient (e.g., "smoking
Simulations (CCS)
cessation," "low-fat diet," "safe-sex techniques"). The
Each Primum case is a dynamic, interactive simulation
order sheet has a free-text entry format; you can type
of a patient-care situation designed to evaluate your
whatever you want. It is not necessary, however, to type
approach to clinical management, including diagnosis,
commands (e.g., "administer," "draw"). The "clerk"
treatment, and monitoring. The cases provide a means
recognizes thousands of different entries typed in
for observing your application of medical knowledge in
different ways. As long as the clerk recognizes the first
a variety of patient-care situations and settings over
three characters of the name or acronym (e.g., "xra,"
varying periods of simulated time. As simulated time
"EKG") you will be prompted for clarification (i.e., you
passes, a patient’s condition may change based on the
will be shown a list of orders beginning with "xra" or the
course of the underlying medical condition(s) or your
acronym "EKG" respectively, including different types
management or both. Patients may present with acute
of x-rays and electrocardiograms).
problems to be managed within a few minutes of
simulated time or with chronic problems to be managed
over several months of simulated time.

Case Interface and Format


You will manage patients using the Primum software.
Information about a patient's condition will be displayed
on the computer screen. At the start of each case, you
will receive a brief description of the reason for the
encounter and the patient’s appearance and status, along
with the vital signs and history. You must initiate
appropriate management and continue care as the
patient's condition changes over simulated time. Patient
information will be provided to you in response to your

25
In some locations (e.g., the office, the inpatient ward), have a few minutes to finalize your orders. You can
there may be cases where a patient is on a medication at cancel orders and add new ones. After finalizing patient
the beginning of the case. In these situations, the care, you will be prompted to enter your final diagnosis.
patient’s current medication will be displayed on the
order sheet (e.g., "oral contraceptives"). These orders If a case has not ended and you feel you are finished
appear with an order time of Day 1 @00:00. You must management of the case, you can end it by advancing
decide whether to continue or cancel the medication, as simulated time. Use the clock as you normally would to
you deem appropriate for the patient’s condition; these receive results of pending tests and procedures. Once
orders remain active throughout the case unless there are no longer any pending patient updates, tests, or
canceled. procedures, continue to use the clock to advance
simulated time until the case ends.
You must advance the clock to see results of tests and
procedures, and to observe effects of treatment. (Note The Patient
that in real life laboratory values fluctuate a small Simulated patients may be from any age group, ethnic or
amount each time they are measured on the same socioeconomic background and may present with well-
patient; successive Primum CCS laboratory test results defined or poorly-defined problems. Patients may
reflect this normal variation. The amount of variation is present with acute or chronic problems or may be
usually very small and should not affect your seeking routine healthcare or health maintenance, with or
interpretation of serial values.) In CCS numeric lab without underlying conditions. Assume that each patient
tests, normal ranges are included with the results. Note you are managing has already given his or her consent
that these normal ranges may differ slightly from those for any available procedure or therapy, unless you
in the MCQ portion of the test. receive a message to the contrary. In the case of a child
or an infant, assume the legal guardians have given
Advancing the clock is what makes "things happen." consent as well.
You select the appropriate clock option after you have
confirmed all the orders you need at a given time. When The Healthcare Network and Facility
there is nothing else you wish to do for a patient, In the Primum CCS healthcare network, you have an
advance the clock to the next time you wish to evaluate outpatient office shared with colleagues across specialty
the patient, check results of previously ordered studies, areas. Your office hours are Monday through Friday
and observe the effect of therapies. As simulated time from 09:00 to 17:00. The hospital facility, a 400-bed
passes, you might receive notification of change in a regional referral center with an emergency department, is
patient's condition through messages from the patient or available 24 hours a day. Standard diagnostic and
the patient's family or from other healthcare providers if therapeutic options are available; no experimental
the patient is in a setting such as the hospital. You decide options are available. The emergency department is a
whether these messages affect your management plan. 24-hour facility and the intensive care unit is available
for medical (including coronary), surgical, obstetric,
Note that if a clock advance to a requested appointment pediatric, and neonatal patients. At the start of each
time is stopped after reviewing results from processed case, you will be informed of the current setting. You
orders, the requested appointment is canceled. Also note should change a patient's location as you deem
that if no results are pending, the case will advance to appropriate.
the next patient update or the end of the case.

Cases end under different circumstances and after


varying amounts of simulated and real time. A case will
end when you reach the maximum allotted real time.
Alternatively, a case may end when you have
demonstrated your skills sufficiently. Encountering the
"End of Case" screen before you think you are finished
managing a patient does not necessarily mean you did
something right or wrong. Once you are prompted with
the "End of Case" screen, real time permitting, you will

26
Surgical and labor/delivery facilities are available, as strategy is to balance efficiency with thoroughness based
well as both inpatient and outpatient laboratory and upon your clinical judgment.
imaging services; however, you cannot transfer patients
to these locations directly. Primum CCS staff will Cost is accounted for indirectly based on the relative
arrange for transfer of patients to these locations for inappropriateness of patient management actions. If you
you. order something that is unnecessary and excessive, your
score will decrease. In considering various options
Evaluative Objectives and Assessment of Your including the location in which you manage the patient,
Performance you need to decide whether the additional cost is
Primum CCS measures those skills a physician employs warranted for better patient care.
in managing a patient over time, with the notable
exception of skills that require human interaction (e.g., Diagnoses and reasons for consultations that you
history taking, physical examination, education and provide when prompted in Primum CCS will not be
counseling, providing emotional support, etc.). Specific used in evaluating your performance at this time, unless
measurement objectives, designed as part of each case needed to investigate unusual test-taking behaviors or
simulation, assess competency in managing a patient response patterns.
with a particular problem or healthcare need in the
context of a specific healthcare setting. The scoring process uses algorithms that represent
codified expert physician policies. These policies allow
The timing and sequencing of indicated actions, as well for wide variations in care protocols among healthcare
as the commission of actions that are not indicated or are settings and systems. The policies are obtained from
potentially harmful, are aggregated in your evaluation. expert physicians who are experienced in training
Individual appropriate patient management actions are physicians and in caring for patients. For each patient
weighted based on degree of appropriateness and may case, the input of expert generalists and specialists is
increase your score by different amounts. Actions that obtained to ensure that performance criteria are
are not indicated and pose greater potential risk to a reasonable for any physician practicing medicine in an
patient decrease your score by greater amounts than unsupervised setting.
actions of lower risk. Seemingly correct management
decisions made in a suboptimal or incorrect sequence or Responsibilities of the Physician
after a delay in simulated time may receive little or no In the simulation, you should function as a primary care
credit. Note that "routine" orders (e.g., diet, ambulation) physician who is responsible for managing each
tend to carry little or no weight in scoring unless they are simulated patient. Management involves addressing a
particularly relevant to the case (e.g., specific diet orders patient's problem(s) and/or concern(s) by obtaining
for a diabetic). diagnostic information, providing treatment, monitoring
patient status and response to interventions, scheduling
Management of patients consistent with widely-accepted appointments and, when appropriate, attending to health
standards of care will achieve a high score, although maintenance screenings and patient education. You will
multiple correct approaches may exist. For example, a manage one patient at a time and should continue to
very efficient approach such as an expert might take manage each patient until the "End of Case" message is
would earn a high score; however, a more thorough displayed.
approach would not necessarily deduct from your score.
Also, taking an innovative but well-documented and Assume that you are the primary care physician for each
accepted approach may achieve the same high score. patient you manage. In this role, you must manage your
Note that in some cases, there may be very little for you patient in both inpatient and outpatient settings.
to do to manage a patient. In those instances, you will be Sometimes this may involve management in several
scored on your ability to recognize situations in which locations—initially caring for a patient in the emergency
the most appropriate action is to refrain from, or defer, department, admitting the patient to the hospital, and
testing and treatment. You will be scored lower if you discharging and following the patient in the outpatient
take an aggressive approach when restraint and setting.
observation are the standard of care. The best overall

27
You should not assume that other members of the
healthcare team (e.g., nurses, consultants) will write or
initiate orders for you. Some routine orders (e.g., "vital
signs" at the beginning of a case and upon change of
location) may be done for you, but you should not make
assumptions regarding other orders. For example, orders
usually requested to monitor a patient's condition, such
as a cardiac monitor and pulse oximetry, are not
automatically ordered. You are responsible for
determining needs and for making all patient
management decisions, whether or not you would be
expected to do so in a real-life situation (e.g., ordering
IV fluids, surgical procedures, or consultations). If you
order a procedure for which you are not trained, the
medical staff in Primum cases will either assist you or
take primary responsibility for implementing your
request.

As in real life, consultants should be called upon as you


deem appropriate. Typically, consultants are not helpful
since the exam is designed to assess your patient
management skills. Nevertheless, you will be evaluated
on whether or not you request the appropriate
consultation when consultation is indicated. For
example, if a surgical procedure is indicated, it may be
appropriate for a primary care physician to request
consultation. However, in some cases it may be
necessary to implement a course of action without the
advice of a consultant or before a consultant is able to
see your patient.

28
Frequently Asked Questions (FAQs)

1. What is Primum® Computer-based Case Simulations (CCS) software?


Primum Computer-based Case Simulations (CCS) software presents an interactive, dynamic simulation of a
patient-care situation designed to evaluate your approach to clinical management, including diagnosis, treatment,
and monitoring. After viewing a description of the patient, initial vital signs, and an initial history, you obtain
diagnostic information and manage the patient until the computer displays a message that the case has ended.

The key features of Primum CCS include:


# simulation of time (e.g., minutes, hours, days, or months)
# health system locations (e.g., you have an office with admitting privileges to a 400-bed tertiary care center)
# free-text entry of orders
# dynamic patient response based on your actions through simulated time

In this uncued testing environment, you have complete responsibility for your patient's care.

2. What are my responsibilities?


No matter what your training or specialty, you should function as a primary care physician and maintain
responsibility for the patient throughout each case. This may involve management in several locations (e.g., initially
caring for a patient in the emergency department, admitting the patient to the hospital, and managing the patient in
the outpatient setting).

You should not assume that other members of the healthcare team (e.g., nurses, medical consultants) will write or
initiate orders for you when a patient is admitted to a facility or transferred for a surgical procedure. You are not
required to write preoperative anesthesia or related orders when someone else is conducting a procedure for you.
However, you should attend to other preparatory patient care that, if neglected, might jeopardize the patient. For
example, in the preoperative setting, this may mean requesting IV fluids, a type and crossmatch, and antibiotics.

In various cases, your duties may include addressing health maintenance issues, handling life-threatening
emergencies, monitoring the effects of treatment, and modifying treatment regimens. The nature of each case
dictates whether or not health maintenance issues are relevant within the simulated time frame.

Your responsibilities to each patient are fulfilled when you see a message indicating the case has ended.

3. How do I manage a patient?


You manage one patient at a time by:
# reviewing the history
# selecting a complete or directed physical examination
# writing orders on the chart
# deciding when, in simulated time, to obtain follow-up history and physical examination or review diagnostic
information by selecting the clock option
# changing the patient's location as you deem appropriate

Based upon information you gather and changes in the patient's condition, you continue to manage the patient
through these options.

29
Since Primum CCS is not designed to assess your ability to complete a history, much of this information is given to
you. You may periodically ask how a patient feels by ordering an interval/follow-up history or monitor the patient
by physical examination. If you believe information is missing from the history or physical examination, assume it
is normal or noncontributory for your patient. Physical examination should be requested if and when you would do
the same with a real patient. Requests for interval history and physical examination automatically advance the clock
in simulated time. To see results of tests and procedures, and to observe effects of treatment, you must advance the
clock. You can write orders before examining a patient; if physical examination reveals findings that you believe
render the orders inappropriate, and the orders have not yet been processed, you can cancel those orders.

The order sheet is the primary means for implementing your patient management plan. You type requests for tests,
procedures, and therapies directly on the order sheet. Each time you confirm orders and want to "make things
happen," use the clock to advance time. When you do so, your orders are implemented, test results are returned, and
therapies are initiated. As you advance the clock, the patient's condition may change based upon the underlying
condition(s) or your management or both.

Note that if a clock advance to a requested appointment time is stopped to review results from processed orders, the
requested appointment is canceled.

Change the patient's location by selecting the Change Location button. You can move the patient from and to home,
office, emergency department, ward, and intensive care unit.

There are some orders in the cases that are not available in every location. If you request a location change with
pending orders that are not available in the new location, you will receive a notification message indicating the
order(s) that will be canceled.

Note that Primum CCS only allows you to manage one patient at a time. Although in real life you order certain
tests or therapies for the relatives or sexual partner of your patient, this option is not available in Primum CCS. It is
possible, however, to order education or counseling for the patient's family or sexual partner. The timing and
sequence of indicated actions, including education and counseling, are evaluated and may affect your score.

4. How do I write/cancel orders?


You write orders by typing your requests, one per line. The Primum "clerk" understands more than 12,000
different terms representing about 2,500 unique orders. As long as the clerk recognizes the first three characters of
the name or acronym (e.g., "xra," "EKG") you will be prompted for clarification (i.e., you will be shown a list of
orders beginning with "xra" or the acronym "EKG" respectively, including different types of x-rays and
electrocardiograms).

If the clerk does not recognize your order, you may have to type it differently. It is not necessary to type commands
(e.g., "administer," "give," "do," "get"); simply type the name of a test, therapy, or procedure (e.g., "chest x-ray,"
"ekg," "pen g," "furosemide," "laparoscopy").

You must request specific drugs by name; the clerk recognizes both generic and trade names. However, the clerk
does not accept class names such as "antacids" or "beta-blockers." You must also specify route and type of
administration (e.g., one-time/bolus or continuous). Assume that "continuous" also encompasses periodic
administration (e.g., every 4 hours) if that is appropriate for the treatment. Note that Intravenous fluids are not
available as a "One Time/ Bolus" order in Primum CCS. Available routes of administration include epidural (EP),
intra-articular (IA), intramuscular (IM), inhalation (IN), intravenous (IV), ophthalmic (OP), otic (OT), oral (PO),
rectal (RE), sublingual (SL), subcutaneous (SQ), topical (TP), and vaginal (VA). It is not necessary to specify
dosages or administration rates; these will not appear on the order sheet, but you can assume these have been
optimized for your patient's condition.

30
To taper a medication, simply discontinue it. If tapering is optimal, it will be done for you. If you decide that you
need to re-order the medication while it is being tapered, assume that the patient has already been tapered from the
medication without adverse consequences.

Medications can not be administered prn. When a medication is indicated for the patient, order it. When it is no
longer indicated, discontinue it.

To discontinue a therapy or cancel a test or procedure, select it on the order sheet and respond "yes" to the prompt.

In some locations (e.g., the office, the inpatient ward), there may be cases where a patient is on a medication at the
beginning of the case. In these situations, the patient's current medication will be displayed on the order sheet (e.g.,
"oral contraceptives"). These orders appear with an order time of Day 1 @00:00. You must decide whether to
continue or cancel the medication, as you deem appropriate for the patient’s condition; these orders remain active
throughout the case unless canceled. The same cancellation steps provided in the previous paragraph also apply to
these orders.

5. What am I supposed to do after I write orders?


After you write orders, you advance the clock to obtain results of diagnostic studies and/or to monitor the patient's
progress. You are not necessarily finished once you make the diagnosis. In many cases, you must initiate treatment,
monitor progress, call consultants, arrange appropriate follow-up, and provide education or other social support.

Once you have managed the patient to your satisfaction, decide when you would like to follow up and advance the
clock to that time. If you can think of no other immediate or future care that is relevant to the patient's current
condition, schedule an appointment for a time when you would like to re-evaluate (e.g., a week, month, or year from
now).

6. Can I change my mind?


You can change your mind at any point in the case by canceling orders and/or writing new orders. However, once
you advance the clock and move forward in simulated time, you cannot go back. As in real life, there is no
opportunity to undo what has already been done. If previously requested actions or delays in appropriate care cause
untoward consequences, your score may be affected adversely.

Discontinue a therapy or cancel a test or procedure by selecting it on the order sheet and responding "yes" to the
prompt.

7. Why are consultants usually not helpful?


Typically consultants are not helpful since the exam is designed to assess your patient management skills.
However, requesting consultation at appropriate times may contribute to your score. Consultants often indicate that
you should initiate treatment in their absence or directly order the surgical procedure you want. In some cases, it
may be necessary to implement a course of action without the advice of a consultant or before a consultant is able
to see your patient. In other cases, a consultant may be helpful after you have obtained enough information to
justify referring the patient to his or her care.

8. What kind of feedback do I get while caring for the patient?


While you care for a patient, you receive results of diagnostic studies you requested and reports of changes in the
patient's condition. (Note that in real life laboratory values fluctuate a small amount each time they are measured on
the same patient; successive Primum CCS laboratory test results reflect this normal variation. The amount of
variation is usually very small and should not affect your interpretation of serial values.) In CCS numeric lab tests,
normal ranges are included with the results. Note that these normal ranges may differ slightly from those in the
MCQ portion of the test.

31
You may obtain intermittent reports about the patient's condition through messages from the patient, the patient's
family, or other healthcare providers. You may also directly request information about the patient's current
condition by ordering interval/follow-up histories.

It is possible that a patient's condition might worsen despite optimal care on your part. It is also conceivable that a
patient's condition might improve with suboptimal care or no care. Scores will be based upon the diagnostic and
therapeutic decisions you make, as well as the timing and sequencing of your actions, and not necessarily on a
patient's final disposition.

Note that interventions ordered at the same time as diagnostic studies will not be reflected in the results.
Interventions don't take effect until an amount of time has passed appropriate for the intervention.

To be certain that a diagnostic test result reflects the intervention, identify the completion time for the intervention
on the order sheet and order the respective diagnostic test at that time. If the completion time is not defined or if the
intervention's effect is gradual (e.g., antibiotics), you must order the diagnostic test at that time when you would
expect a clinical effect.

9. How long do cases last?


Cases can last from a few minutes to several months of simulated time. You are not told how much simulated time
will elapse in each case. It is your responsibility to manage simulated time based upon your understanding of the
urgency of the case.

The real time allotted to manage each patient may vary with the type of case and your actions. You will be allotted
a maximum of 25 minutes per case, but you may not need to use the entire time. For example, if you accomplish a
case's measurement objectives quickly, it may end in a few minutes. Before you begin each case in the examination,
you will be informed of the maximum time allotted.

If, during the examination, you do not use all the allotted real time for a case the "remaining" real time is not added
to the allotted real time for any other case.

10. How do I know when I have finished a case?


Near the end of each case, you will be warned that the case is ending shortly. At that time, you will be given a few
minutes to cancel existing orders and/or write new orders for the immediate or future care of problems related to the
patient's current condition. Once you confirm these orders, you will be prompted to enter a diagnosis and will then
receive an "END OF CASE" message.

If a case has not ended and you feel you are finished management of the case you can end it by advancing
simulated time. Use the clock as you normally would to receive results of pending tests and procedures. Once there
are no longer any pending patient updates, tests, or procedures, continue to use the clock to advance simulated time
until the case ends.

11. Does computer experience matter?


Assuming that you take the time to familiarize yourself with the basic operations of the computer (e.g., use of the
keyboard, mouse, etc.), computer experience should not affect your performance. However, experience and practice
with Primum cases can have an impact. It is essential that you become familiar with both the software interface and
the background information provided. Experience has shown that those who do not practice with the format and
mechanics of managing the patients in Primum CCS are likely to be at a disadvantage when taking the cases under
standardized test conditions.

12. How is my performance scored?


The timing and sequencing of indicated actions, as well as the commission of actions that are not indicated or are
potentially harmful, are aggregated in your evaluation. Individual appropriate patient management actions are

32
weighted based on degree of appropriateness and may increase your score by different amounts. Actions that are
not indicated and pose greater potential risk to a patient decrease your score by greater amounts than actions of
lower risk. Seemingly correct management decisions made in a suboptimal or incorrect sequence or after a delay in
simulated time may receive little or no credit.

Note that the importance of the timeliness of your actions varies in nonurgent cases; your score may be affected by
the timeliness of your response based on the case. "Routine" orders (e.g., diet, ambulation) tend to carry little or no
weight in scoring unless they are particularly relevant to the case (e.g., specific diet orders for a diabetic).

Management of patients consistent with widely-accepted standards of care will achieve a high score, although
multiple correct approaches may exist. For example, a very efficient approach such as an expert might take would
earn a high score; however, a more thorough approach would not necessarily deduct from your score. Also, taking
an innovative but well-documented and accepted approach may achieve the same high score. Note that in some
cases, there may be very little for you to do to manage a patient. In those instances, you will be scored on your
ability to recognize situations in which the most appropriate action is to refrain from, or defer, testing and treatment.
You will be scored lower if you take an aggressive approach when restraint and observation are the standard of
care. The best overall strategy is to balance efficiency with thoroughness based upon your clinical judgment.

Cost is accounted for indirectly based on the relative inappropriateness of patient management actions. If you order
something that is unnecessary and excessive, your score will decrease. In considering various options including the
location in which you manage the patient, you need to decide whether the additional cost is warranted for better
patient care. Diagnoses and reasons for consultations that you provide when prompted in Primum CCS will not be
used in evaluating your performance at this time, unless needed to investigate unusual test-taking behaviors or
response patterns.

The scoring process uses algorithms that represent codified expert physician policies. These policies allow for wide
variations in care protocols among healthcare settings and systems. The policies are obtained from expert
physicians who are experienced in training physicians and in caring for patients. For each patient case, the input of
expert generalists and specialists is obtained to ensure that performance criteria are reasonable for any physician
practicing medicine in an unsupervised setting.

13. Are there differences in practice and live case functionality?


There are no differences between case functionality with the practice Primum Computer-based Case Simulations
(CCS) software and the cases on the examination. However, there are several differences related to how cases are
presented in practice and how they are presented in the examination. These differences are summarized below.
# In the practice session, there is the option to choose whether to run blocks of untimed cases or a block of timed
cases. During the examination, the cases are presented one at a time with a specified and limited amount of real
time indicated for each case.
# In the event of a computer problem during a live examination, a case simulation may be restarted by the testing
center staff. Only one restart per case is permitted. If a case is restarted more that once, the restart restriction
will prevent the interrupted case simulation from being completed and the next case will appear.
# During the examination it may take longer to process history and physical exam requests; order tests, therapies,
or procedures; advance the clock; and change location. This is due to increased network computer resource
requirements on the examination.
# Prior to the start of each case in the examination a screen is displayed indicating the amount of real time
allotted for that case.
# After completion of each case during the examination, a screen is displayed that asks if the examinee would like
to take a break.

14. What do I do if the scroll bar "freezes"?

33
If the scroll bar "freezes," press the keyboard Page Up or Page Down buttons to scroll up or down and review
information. If this solution does not work during the live examination, contact a proctor.

34
Sample Step 3 Questions

Sample Questions

The following pages include 144 sample test questions. These questions are the same as those you install on
your computer from the USMLE website or CD. For information on obtaining the test software and
additional information on preparing to take the test and testing, you must review the 2003 USMLE Bulletin
of Information: see Preparing for the Test (pages 9–10) and Testing (pages 16–19). Please note that
reviewing the sample questions as they appear on pages 38–85 is not a substitute for acquainting yourself
with the test software. You should run the Step 3 tutorial and sample test questions that are provided on the
USMLE website or CD well before your test date.

These sample questions are illustrative of the types of questions used in the Step 3 examination. Although
the questions exemplify content on the examination, they may not reflect the content coverage on individual
examinations. Questions are grouped together by the setting in the same manner as in the actual computer-
administered test blocks. In the actual examination, the questions will be presented one at a time in a format
designed for easy on-screen reading, including use of exhibit buttons (separate windows) for the Normal
Laboratory Values Table (included here on pages 35–36) and some pictorials. Photographs, charts, and x-ray
films referred to in this booklet are not of the same quality as the pictorials used in the actual examination.
In addition, you will have the capability to adjust the brightness and contrast of pictorials on the computer
screen.

To take the following sample test questions as they would be timed in the actual examination, you should
allow a maximum of 1 hour for each 48-item block, and a maximum of 30 minutes for each 24-item block,
for a total of 3 hours. Please be aware that most examinees perceive the time pressure to be greater during
an actual examination. An answer sheet for recording answers is provided on page 37. In the actual
examination, answers will be selected on the screen; no answer sheet will be provided. An answer key is
provided on page 86.

35
USMLE STEP 3 LABORATORY VALUES

* Included in the Biochemical Profile

BLOOD, PLASMA, SERUM REFERENCE RANGE SI REFERENCE INTERVALS


* Alanine aminotransferase (ALT), serum ................ 10-40 U/L......................................................... 10-40 U/L
* Alkaline phosphatase, serum .................................. Male: 30-100 U/L ............................................. Male: 30-100 U/L
Female: 45-115 U/L ......................................... Female: 45-115 U/L
Amylase, serum ...................................................... 25-125 U/L ....................................................... 25-125 U/L
* Aspartate aminotransferase (AST), serum ............... 15-40 U/L.......................................................... 15-40 U/L
* Bilirubin, serum (adult), total // direct .................... 0.1-1.0 mg/dL // 0.0-0.3 mg/dL ........................ 2-17 µmol/L // 0-5 µmol/L
Calcium, serum (total) ............................................ 8.4-10.2 mg/dL ................................................. 2.1-2.8 mmol/L
* Cholesterol, serum
Total..................................................................... 150-240 mg/dL ................................................. 3.9-6.2 mmol/L
HDL..................................................................... 30-70 mg/dL...................................................... 0.8-1.8 mmol/L
LDL...................................................................... <160 mg/dL....................................................... <4.2 mmol/L
Cortisol, serum ....................................................... 8:00 AM: 5-23 µg/dL // 4:00 PM: 3-15 µg/dL .... 138-635 nmol/L // 82-413 nmol/L
8:00 PM: # 50% of 8:00 AM .............................. Fraction of 8:00 AM: # 0.50
Creatine kinase, serum ........................................... Male: 25-90 U/L ............................................... 25-90 U/L
Female: 10-70 U/L ........................................... 10-70 U/L
* Creatinine, serum ................................................... 0.6-1.2 mg/dL ................................................... 53-106 µmol/L
Electrolytes, serum
* Sodium (Na+) ........................................................ 135-146 mEq/L ................................................. 135-146 mmol/L
* Potassium (K+) ...................................................... 3.5-5.0 mEq/L ................................................... 3.5-5.0 mmol/L
* Chloride (C1-) ....................................................... 95-105 mEq/L ................................................... 95-105 mmol/L
* Bicarbonate (HCO3-) ............................................ 22-28 mEq/L ..................................................... 22-28 mmol/L
Ferritin, serum ........................................................ Male: 15-200 ng/mL ......................................... 15-200 µg/L
Female: 12-150 ng/mL ..................................... 12-150 µg/L
Follicle-stimulating hormone, serum/plasma ......... Male: 4-25 mIU/mL ......................................... 4-25 U/L
Female: premenopause 4-30 mIU/mL .............. 4-30 U/L
midcycle peak 10-90 mIU/mL ...................... 10-90 U/L
postmenopause 40-250 mIU/mL .................. 40-25 U/L
Gases, arterial blood (room air)
PO2 ......................................................................... 75-100 mm Hg .................................................. 10.0-14.0 kPa
PCO2 ....................................................................... 35-45 mm Hg .................................................... 4.4-5.9 kPa
pH ......................................................................... 7.35-7.45 ........................................................... [H+] 36-44 nmol/L
* Glucose, serum ....................................................... Fasting: 70-110 mg/dL ...................................... 3.8-6.1 mmol/L
2-h postprandial: < 120 mg/dL ..................... < 6.6 mmol/L
Immunoglobulins, serum
IgA ...................................................................... 76-390 mg/dL ................................................... 0.76-3.90 g/L
IgE ...................................................................... 0-380 IU/mL ..................................................... 0-380 kIU/L
IgG ...................................................................... 650-1500 mg/dL ............................................... 6.5-15 g/L
IgM ..................................................................... 40-345 mg/dL ................................................... 0.4-3.45 g/L
Iron ......................................................................... 50-170 µg/dL .................................................... 9-30 µmol/L
Lactate dehydrogenase, serum ................................ 45-90 U/L ........................................................ 45-90 U/L
Luteinizing hormone, serum/plasma ...................... Male: 6-23 mIU/mL ......................................... 6-23 U/L
Female: follicular phase 5-30 mIU/mL ............. 5-30 U/L
midcycle 75-150 mIU/mL ............................ 75-150 U/L
postmenopause 30-200 mIU/mL .................. 30-200 U/L
Osmolality, serum .................................................. 275-295 mOsmol/kg H2O ................................. 275-295 mOsmol/kg H2O
Phosphorus (inorganic), serum ............................... 3.0-4.5 mg/dL ................................................... 1.0-1.5 mmol/L
Proteins, serum
Total (recumbent) ................................................. 6.0-7.8 g/dL ...................................................... 60-78 g/L
Albumin ................................................................ 3.5-5.5 g/dL ...................................................... 35-55 g/L
Globulin .............................................................. 2.3-3.5 g/dL ...................................................... 23-35 g/L
Thyroid-stimulating hormone (TSH), serum .......... 0.5-5.0 µU/mL .................................................. 0.5-5.0 mU/L
Thyroxine (T4), serum ............................................ 5-12 µg/dL ........................................................ 64-155 nmol/L
Triglycerides........................................................... 35-160 mg/dL .................................................... 0.4-1.81 mmol/L
Triiodothyronine (T3) resin uptake ......................... 25-35% ............................................................. 0.25-0.35
* Urea nitrogen, serum (BUN) .................................. 7-18 mg/dL ....................................................... 1.2-3.0 mmol/L
Uric acid, serum ..................................................... 3.0-8.2 mg/dL ................................................... 0.18-0.48 mmol/L
LABORATORY VALUES (continued)

CEREBROSPINAL FLUID REFERENCE RANGE SI REFERENCE INTERVALS


Cell count .................................................................... 0-5 cells/mm3 ...................................... 0-5 x 106/L
Chloride ...................................................................... 118-132 mEq/L .................................. 118-132 mmol/L

36
Gamma globulin .......................................................... 3-12% total proteins .......................... 0.03-0.12
Glucose ........................................................................ 40-70 mg/dL ....................................... 2.2-3.9 mmol/L
Pressure ....................................................................... 70-180 mm H2O ................................. 70-180 mm H2O
Proteins, total ............................................................... < 40 mg/dL ......................................... < 0.40 g/L

HEMATOLOGIC
Bleeding time (template) ............................................. 2-7 minutes ........................................ 2-7 minutes
CD4 cell count ................................................................ > 500/mm3
Erythrocyte count .......................................................... Male: 4.3-5.9 million/mm3 ............... 4.3-5.9 x 1012/L
Female: 3.5-5.5 million/mm3 ............ 3.5-5.5 x 1012/L
Erythrocyte sedimentation rate (Westergren) ................ Male: 0-15 mm/h ............................... 0-15 mm/h
Female: 0-20 mm/h ........................... 0-20 mm/h
Hematocrit ..................................................................... Male: 41-53% .................................... 0.41-0.53
Female: 36-46% ................................ 0.36-0.46
Hemoglobin, blood ........................................................ Male: 13.5-17.5 g/dL ......................... 2.09-2.71 mmol/L
Female: 12.0-16.0 g/dL ..................... 1.86-2.48 mmol/L
Hemoglobin Alc ............................................................ # 6% ................................................... # 0.06%
Leukocyte count and differential
Leukocyte count .......................................................... 4500-11,000/mm3 .............................. 4.5-11.0 x 109/L
Neutrophils, segmented ............................................... 54-62% .............................................. 0.54-0.62
Neutrophils, band ........................................................ 3-5% .................................................. 0.03-0.05
Eosinophils .................................................................. 1-3% .................................................. 0.01-0.03
Basophils ..................................................................... 0-0.75% ............................................. 0-0.0075
Lymphocytes ............................................................... 25-33% .............................................. 0.25-0.33
Monocytes ................................................................... 3-7% .................................................. 0.03-0.07
Mean corpuscular hemoglobin (MCH) .......................... 25-35 pg/cell ..................................... 0.39-0.54 fmol/cell
Mean corpuscular hemoglobin
concentration (MCHC) ................................................ 31-36% Hb/cell ................................. 4.81-5.58 mmol Hb/L
Mean corpuscular volume (MCV) ................................. 80-100 µm3 ..................................... 80-100 fl
Partial thromboplastin time (activated) ......................... < 28 seconds ...................................... < 28 seconds
Platelet count ................................................................. 150,000-400,000/mm3 ..................... 150-400 x 109/L
Prothrombin time ........................................................... < 12 seconds ...................................... < 12 seconds
Reticulocyte count ......................................................... 0.5-1.5% of red cells ......................... 0.005-0.015
Volume
Plasma .......................................................................... Male: 25-43 mL/kg ........................... 0.025-0.043 L/kg
Female: 28-45 mL/kg ........................ 0.028-0.045 L/kg
Red cell ........................................................................ Male: 20-36 mL/kg ........................... 0.020-0.036 L/kg
Female: 19-31 mL/kg ....................... 0.019-0.031 L/kg

URINE
Calcium .......................................................................... 100-300 mg/24 h ............................... 2.5-7.5 mmol/24 h
Creatinine clearance...................................................... Male: 97-137 mL/min
Female: 88-128 mL/min
Osmolality ...................................................................... 50-1400 mOsmol/kg H2O
Oxalate ........................................................................... 8-40 µg/mL ....................................... 90-445 µmol/L
Proteins, total ................................................................. < 150 mg/24 h ................................... < 0.15 g/24 h

BODY MASS INDEX Rec=19-25 kg/m2

37
Answer Form for Step 3 Sample Questions

Block 1

1. 9. 17. 25. 33. 41.


2. 10. 18. 26. 34. 42.
3. 11. 19. 27. 35. 43.
4. 12. 20. 28. 36. 44.
5. 13. 21. 29. 37. 45.
6. 14. 22. 30. 38. 46.
7. 15. 23. 31. 39. 47.
8. 16. 24. 32. 40. 48.

Block 2

49. 57. 65. 73. 81. 89.


50. 58. 66. 74. 82. 90.
51. 59. 67. 75. 83. 91.
52. 60. 68. 76. 84. 92.
53. 61. 69. 77. 85. 93.
54. 62. 70. 78. 86. 94.
55. 63. 71. 79. 87. 95.
56. 64. 72. 80. 88. 96.

Block 3

97. 101. 105. 109. 113. 117.


98. 102. 106. 110. 114. 118.
99. 103. 107. 111. 115. 119.
100. 104. 108. 112. 116. 120.

Block 4

121. 125. 129. 133. 137. 141.


122. 126. 130. 134. 138. 142.
123. 127. 131. 135. 139. 143.
124. 128. 132. 136. 140. 144.

38
Sample Step 3 Questions

GENERAL INSTRUCTIONS: Read each question carefully and in the order in which it is presented. Then select the one best response
option of the choices offered. There may be either 4 or 5 response options. More than one option may be partially correct. You must select
the ONE BEST answer and fill in the corresponding circle on the answer sheet.

Some items are grouped together around a clinical vignette as a set or case; be particularly careful to read and answer these cases or sets of
items in the order they are presented.

The items in this exam are divided among the clinical settings:

Block 1 Office/Health Center Items 1-48


Block 2 Office/Health Center Items 49-96
Block 3 Inpatient Facilities Items 97-120
Block 4 Emergency Department Items 121-144

Block 1: Office/Health Center


Items 1-48; Time - 60 minutes

You see patients in two locations: your office suite, which is adjacent to a hospital, and at a community-based health center. Your
office practice is in a primary care generalist group. Patients are seen for routine and urgent care at the office and health center.
Most of the patients you see are from your own practice, although occasionally you will see a patient cared for by one of your
associates and reference may be made to the patient's medical records. Known patients may be managed by telephone, and you
may have to respond to questions about information appearing in the public media, which will require interpretation of the
medical literature. The laboratory and radiology departments have a full range of services available.

ALL ITEMS REQUIRE SELECTION OF ONE BEST CHOICE.

1. A 21-year-old man comes to the clinic because he has become increasingly short of breath and has had a cough for the past week.
He appears dyspneic and has a temperature of 38.3EC (101.0EF). On physical examination he has bibasilar rales and generalized
lymphadenopathy (1 to 2 cm). Rectal examination shows multiple perianal contusions and a small amount of blood oozing from the
anal orifice. Chest x-ray film shows bilateral patchy alveolar infiltrates. The most appropriate course of action is to order blood tests
and to

(A) admit him to the hospital and begin administration of trimethoprim-sulfamethoxazole, intravenously
(B) admit him to the hospital and begin administration of penicillin and gentamicin, intravenously
(C) begin administration of erythromycin, orally, and see him again the next day
(D) prescribe isoniazid and rifampin, orally
(E) recommend aspirin, fluids and rest at home

2. A 17-year-old white girl returns to the health center for a family planning follow-up visit. She gave birth to a healthy baby girl 8
months ago and does not want to become pregnant again. She is monogamous with the father of her baby but worries that he is not
monogamous with her. They live together and are not married. She is taking an oral contraceptive and "sometimes" uses condoms.
She says that she has great trouble remembering to take her pills and wants to discuss other contraceptive options. In addressing this
issue, which of the following is the most appropriate next step?

(A) Advise her to continue taking an oral contraceptive because it is one of the most effective methods of birth control
(B) Advise her that an intrauterine device would be a good contraceptive choice for her
(C) Discuss implantable or injection long-term progestational contraceptive agents
(D) Discuss the option of tubal ligation
(E) Fit the patient with a diaphragm and instruct her on proper use

39
3. A 19-year-old African-American college student comes to the student health center because of marked fatigue. Temperature is
38.3°C (101.0°F). Physical examination shows striking pallor of skin, nail beds and conjunctivae. There are petechial hemorrhages
in the skin of his legs. A soft, blowing systolic murmur is present over the precordium. No other abnormalities are present. The
most appropriate study at this time is

(A) complete blood count


(B) determination of bleeding and clotting time
(C) examination of bone marrow aspirate
(D) hemoglobin electrophoresis
(E) serological testing for infectious mononucleosis

4. A 12-year-old girl is referred to the clinic by the school nurse for evaluation of scoliosis. The girl's scoliosis was detected during a
routine screening examination at the school, and it appears to be mild (curve less than 10 degrees). She is athletic and is otherwise
in good health. During the physical examination, particular attention should be given to which of the following?

(A) Arm length


(B) Blood pressure
(C) Body weight
(D) Cardiac examination
(E) Stage of pubertal development

5. A 10-month-old infant is brought to the health center by his mother because of eight watery bowel movements during the past
24 hours. On physical examination the infant is lethargic and somnolent. The mucous membranes are dry and the skin turgor is
poor. The anterior fontanel is sunken. At his last visit 2 weeks ago, he weighed 10 kg (22 lb); today his weight is 9 kg (20 lb). His
temperature is 37.2EC (99.0EF), pulse is 170/min and blood pressure is 100/60 mm Hg. Which of the following is the most
appropriate next step in management of this infant?

(A) Begin intravenous hydration with isotonic saline solution


(B) Educate the mother on how to recognize dehydration and worsening clinical symptoms in her infant
(C) Obtain serum electrolyte concentrations and begin oral rehydration therapy
(D) Obtain a stool culture and begin amoxicillin therapy
(E) Tell the mother to stop breast-feeding and have her give the infant a soy-based formula

40
Items 6-7

A 45-year-old woman is brought to the health center by her husband because of nausea, confusion, chills, fever, flank pain and cloudy
urine. She has a history of insulin-dependent diabetes mellitus, poorly controlled hypertension and recurrent urinary tract infections. Her
vital signs are:

Temperature 40.0°C (104.0°F)


Pulse 120/min
Respirations 24/min
Blood pressure 110/70 mm Hg

Funduscopic examination shows diabetic retinopathy, which is unchanged from the previous examination. Neck is supple. Lungs are clear
to auscultation and percussion. Examination of the abdomen is normal. Marked pain is present at the right costovertebral angle. Several
hemorrhagic bullous lesions are noted on the extremities.

6. Without prompt and aggressive treatment, this patient is most likely to develop which of the following?

(A) Diabetic ketoacidosis


(B) Hyperosmolar coma
(C) Meningitis
(D) Pneumonia
(E) Septic shock

7. If this patient were to develop anuria, which of the following would be the most likely cause?

(A) Acute papillary necrosis


(B) Bladder outlet obstruction
(C) Neurogenic bladder
(D) Renal lithiasis
(E) Tumor encroachment on the ureters

END OF SET

8. A 23-year-old registered nurse comes to the employee health clinic because she says, "I'm too tired to work." She has had
increasing fatigue, malaise and anorexia during the past several days. Laboratory studies show:

Serum Blood
ALT 1160 U/L PT 13 sec
Bilirubin 1.8 mg/dL
HBsAg Positive

She is instructed to rest at home and return in 3 days if no new symptoms develop. Two days after the visit she calls to say that she
has now developed an urticarial rash and swelling of the joints of her fingers. At this time which of the following is the most correct
statement about her condition?

(A) The arthritis and rash are the result of an associated immune complex disorder
(B) The arthritis and rash are unrelated to her liver disease
(C) It is unlikely that her blood is infectious
(D) She has a 50% risk for developing chronic liver disease
(E) She should be given hepatitis B immune globulin

41
9. A 47-year-old Italian-American man comes to the office for the first time for routine medical care. He has been referred to you by
his psychiatrist who has informed you that the patient has paranoid personality disorder. The patient has no other medical problems
at this time. He is unmarried, lives alone and has no close friends, but he occasionally attends family gatherings. He functions well
working alone in a technical position in an engineering firm. Which of the following is the best way to structure the
physician-patient relationship with this patient?

(A) Avoid giving him excessive details about possible, but infrequent, side effects and complications in order to avoid
triggering his paranoia
(B) Explain the rationale for any diagnostic procedures and treatment regimens in some detail, adopting a professional,
but not overly friendly stance
(C) Go out of your way to be warm and friendly so that he can develop trust in you
(D) Have his psychiatrist, with whom he has been working for several years, take the lead in presenting medical treatment
options
(E) Try to communicate with his family or medical personnel when he is not present in order to overcome his withholding
information because of distrust

10. A 12-month-old boy is brought to the office by his mother for his routine health check-up. She informs you that she has just been
diagnosed with hypercholesterolemia. Her fasting serum total cholesterol concentration was 260 mg/dL and her LDL-cholesterol
concentration was 130 mg/dL (rec<129 mg/dL). She is unaware of a family history of coronary artery disease because she was
raised by her godmother when her parents died in their early 30s in a motor vehicle accident. A special diet has been recommended
for her; however, she is very concerned about the risk of hypercholesterolemia for her son. The most appropriate management at
this time is to

(A) ask her to reduce the child's fat intake and give him skim milk instead of whole milk
(B) do nothing until the child is 2 years of age
(C) have the child return for a fasting lipoprotein analysis
(D) obtain a random serum total cholesterol concentration for the child today
(E) refer the child to a lipid specialist

11. A 75-year-old white man comes to the office because of increasing shortness of breath on exertion for the past 2 to 3 months. He has
a history of hypertension for which he takes hydrochlorothiazide. On physical examination his pulse is 80/min. There is an apical
lift displaced to the left and a harsh diastolic murmur at the base. Further physical examination is most likely to show

(A) bifid pulse


(B) low-amplitude pulse
(C) pulsus alternans
(D) pulsus paradoxus
(E) wide pulse pressure

12. A 33-year-old white woman asks you for a third opinion because two other physicians have been unsuccessful in alleviating her
multiple symptoms. She complains of numbness in her face and a pain deep behind her left eye. She describes weakness in her
upper extremities and a "clumsy right hand." She says she intermittently wets her pants. She relates her history in an emotional
fashion and emphasizes that the symptoms are all worse when the weather is hot. Physical examination discloses increased deep
tendon reflexes in both biceps and triceps. There is temporal pallor and partial atrophy of the left optic nerve head. Which of the
following studies will most likely confirm the diagnosis?

(A) Cerebrospinal fluid analysis


(B) Electromyography
(C) Minnesota Multiphasic Personality Inventory
(D) MRI of the head
(E) Serum protein electrophoresis

42
13.

The lesion shown is of a 1-year-old girl who is brought to the office by her parents because they want a second opinion regarding the
growth on her leg. The mother notes that it was small and flat at birth and has grown thicker in the past 6 months. The mother says,
"My usual physician seems unconcerned, but my mother-in-law says it looks awful and that something should be done." The child is
otherwise healthy. At this time it is most appropriate to tell the parents that the best cosmetic result will occur with which of the
following?

(A) Injection with a corticosteroid


(B) Observation only
(C) Radiation therapy
(D) Surgical excision
(E) Topical application of a corticosteroid

14. A 15-year-old boy comes to the office for a sports participation physical examination. He has been playing in a summer basketball
league and now wants to try out for the high school team. His last physical examination was 2 years ago and, according to him, he
has been healthy except for a cold 2 weeks ago. Before you begin the physical examination, the nurse informs you that his routine
urinalysis shows:

Color Tea-colored/dark WBC 7/hpf


Specific gravity 1.030 RBC >100/hpf, a few red cell casts
pH 5.5 Bacteria Negative
Protein 2+ Glucose Negative
Ketones Negative

These laboratory results are most indicative of which of the following?

(A) Cystitis
(B) Glomerulonephritis
(C) Nephrotic syndrome
(D) Pyelonephritis
(E) Renal calculi

43
15. A 16-year-old Anglo-American girl comes to the office because she has missed a menstrual period and a home pregnancy test was
positive. Bimanual examination discloses an enlarged uterus, and a urine pregnancy test is positive. She is estimated to be
approximately 6 to 8 weeks pregnant. You have been the family's physician since she was born. She asks that you please not tell her
parents. Which of the following is the most appropriate immediate response?

(A) Attempt to persuade her to share the information with at least one adult member of her family
(B) Explain that by law you are unable to maintain confidentiality
(C) Explore with her the reasons for her not feeling comfortable sharing the information with her parents
(D) Point out that she will not be able to hide her pregnancy for very long
(E) Reassure her, but notify her mother after she leaves

16. During a routine physical examination of a 2-week-old Jewish neonate, a grade 3/6 early systolic murmur is heard. The mother
reports no signs or symptoms of illness in the baby. The neonate is otherwise asymptomatic. Physical examination at birth was
normal. The most appropriate next step is to

(A) do funduscopic examination


(B) measure arterial blood pressures in the arms and legs
(C) order abdominal ultrasonography
(D) order chest x-ray film
(E) order electrocardiography

17. A 29-year-old Italian-American woman comes to the office for her first prenatal visit. Her last menstrual period was 16 weeks ago.
This is her first pregnancy; her family history is unremarkable. She has heard that people of Mediterranean ancestry are at risk for
carrying a gene for β-thalassemia. She asks to be tested for this. Which of the following is the most appropriate initial diagnostic
study?

(A) Complete blood count with red cell indices


(B) Hemoglobin electrophoresis
(C) Red cell osmotic fragility test
(D) Restriction-fragment length polymorphism (RFLP) analysis of her β-globin gene
(E) Reticulocyte count

18. A 22-year-old man returns to the office for a follow-up visit 4 weeks after being diagnosed with mononucleosis. He is a college
football player and the football season is just beginning. At his initial visit, he had a sore throat, nausea and a temperature of
39.4EC (103.0EF). Physical examination at that time showed pharyngitis, submandibular and posterior cervical lymphadenitis, and
splenomegaly. Complete blood count (CBC) done at that time showed a leukocyte count of 18,000/mm3 with an elevated number of
monocytes, and a Monospot® test was positive. The course of his illness has been uncomplicated, but today he still complains of
some residual fatigue. Before you examine him, he asks you, "When can I play again?" Which of the following is the most
appropriate response to the patient regarding when he may return to contact sports?

(A) Next season


(B) When a Monospot® test is negative
(C) When he is no longer symptomatic
(D) When his CBC is normal
(E) When his physical examination is normal

44
Items 19-20

A 34-year-old man comes to the office because he has experienced 6 weeks of gradually increasing fevers, dry cough and shortness of
breath. Records show he has lost 5.4 kg (12 lb) since his last routine visit 6 months ago. Today, on physical examination, there are several
large, nontender, anterior and posterior cervical lymph nodes. His medical history includes infectious mononucleosis, hepatitis B,
impetigo, prostatitis and tinea versicolor.

19. Which of the following factors in his history is most suggestive of pneumonia related to a sexually transmitted disease?

(A) Hepatitis B
(B) Impetigo
(C) Infectious mononucleosis
(D) Prostatitis
(E) Tinea versicolor

20. Which of the following is the most appropriate question to ask this patient when inquiring about his sexual orientation?

(A) "Have you been sexually active with men, women, or both?"
(B) "Have you ever had homosexual relations?"
(C) "How many girlfriends have you had?"
(D) "How many times have you had sex with other men?"
(E) "What are your views about homosexuality?"
END OF SET

21. A male colleague asks you to write a prescription for a narcotic analgesic for one of his female patients. You have noticed that this
patient frequently has been coming by the office to see your colleague, and that several of the visits have been marked "No Charge."
When you ask your colleague why he cannot write the prescription himself, he seems defensive and says, "because I don't want
anybody to get the wrong idea." Which of the following is the most appropriate response to your colleague?

(A) "It sounds like there is more to this story than you are telling me; maybe we should talk about it."
(B) "I wish I could help you, but I never prescribe that medication for a patient unless I have seen the patient myself."
(C) "I will do this for you once, but I will need to see your patient in the office before I can write another prescription."
(D) "Maybe I should see your patient in the office myself, and then decide if she needs the medication."
(E) "You have seemed a little nervous lately. You aren't getting in over your head, are you?"

45
22. A 26-year-old man whom you have treated with alprazolam for anxiety contacts you via the answering service concerning his 20-
year-old sister. You have seen his sister in your practice for routine family planning and preventive care. He speaks excitedly, and
says, "My sister is just lying on the floor moaning. She fell on the floor while we were eating dinner. There wasn't any reason for it,
except the room was very hot from the cooking." According to her brother, she slowly slumped over onto her husband while they
were sitting at the dinner table. The family then eased her onto the floor. She denies any pain and speaks in complete sentences. Her
brother sees no rash and no evidence of bruising. The patient looks at him when she is talking, but she intermittently turns away,
moving her head from side to side and moaning and crying. He also tells you that his sister and her husband kept an appointment
with their clinical psychologist today for marriage counseling. According to the information available, which of the following is the
most likely cause of this behavior?

(A) Acute dystonic reaction


(B) Hyperthermia
(C) Psychogenic
(D) Toxic shock syndrome
(E) Vasovagal syncope

23. A 22-year-old woman returns to the office for a follow-up visit. She was last seen 2 weeks ago because of a facial rash that worsens
with sun exposure. Today she says she has joint pain. She says, "It just hurts everywhere. Sometimes my knees hurt, and sometimes
it’s my elbows or my ankles. And I have been feeling tired all the time. I just don't have any energy." She denies any joint swelling.
An office urinalysis shows 3+ protein. You consider the diagnosis of systemic lupus erythematosus and plan additional diagnostic
testing. If she has systemic lupus erythematosus, studies are most likely to show which of the following?

(A) Decreased numbers of helper T cells


(B) Decreased serum concentrations of C3 and C4
(C) Increased numbers of plasma cells
(D) Serum antimicrosomal antibodies
(E) Serum antiplatelet antibodies

46
Items 24-25

A 59-year-old woman comes to the office because of pain in the left lower quadrant of her abdomen for the past 48 hours. The pain is
associated with tenesmus, loss of appetite and a sensation of being febrile. She has a history of long-standing irritable bowel symptoms and
has not taken any pharmacotherapy. Vital signs are: temperature 37.3EC (99.2EF), pulse 84/min and regular and respirations 12/min. On
physical examination the abdomen is tender in the left lower quadrant; there is no rebound tenderness and bowel sounds are normal.
Leukocyte count is 10,200/mm3 with 71% segmented neutrophils and 3% band forms. The patient is sent home on a liquid diet and
tetracycline therapy. Three days later she returns because the pain has become worse and she is now having chills.

24. Which of the following is the most appropriate next step?

(A) Admit her to the hospital and begin cefoxitin therapy, intravenously
(B) Admit her to the hospital and prepare her for an immediate operation
(C) Change to ciprofloxacin therapy, orally, and send her home
(D) Continue the present course of therapy
(E) Do colonoscopy

25. Following the patient's recovery from the acute phase of this illness, which of the following is the most appropriate long-term
management?

(A) Annual endoscopic examination for neoplasia


(B) High-fiber diet
(C) Low-residue diet
(D) Reassurance only
(E) Use of prophylactic antibiotic therapy for 1 week each month
END OF SET

26.

A husband and wife, ages 29 years and 34 years, respectively, come to the office for advice regarding their risk for having hearing-
impaired children. They are both hearing-impaired and require hearing aids. Their hearing loss is sensorineural and is not
associated with any other health problems. The wife tells you, "We have both learned to live with this disability, but we want to take
it into account before we decide to have children." Their pedigree is shown with the patients identified as II2 and II3. You advise
them that:

(A) Accurate risk estimation is impossible without further evaluation


(B) Because they are hearing-impaired, all their children will be hearing-impaired
(C) It is unlikely that they will have hearing-impaired children
(D) Only their male children will be hearing-impaired
(E) They can have amniocentesis during pregnancy to test whether the infant will be hearing-impaired

47
Items 27-29

An 18-year-old white high school student comes to the office in late August because of a stuffy nose for 1 week. He reminds you that he
has had severe hay fever in the fall for the past 10 years. Review of his chart shows that he has positive skin tests to ragweed, dust and dust
mites, and he is receiving maintenance immunotherapy with extracts of these antigens. He also takes over-the-counter antihistamines for
symptomatic relief. This regimen has not provided relief so far this season. Physical examination is normal, apart from clear rhinorrhea.
He is afebrile, and there is normal transillumination of the frontal and maxillary sinuses. In reviewing his medical records you note that
smears of his nasal mucus contained large numbers of eosinophils. There is no record of any extensive immunologic work-up. You tell
him that he now has either an early viral upper respiratory tract infection or the beginnings of his seasonal allergic rhinitis. You suggest
that he use a corticosteroid nasal spray plus his usual antihistamines as needed. The appropriate treatment is undertaken, but 4 days later
he returns because of a toothache and fever. On physical examination he has right facial fullness and pain below his eye when he leans
forward. There is tenderness in the region of the upper premolar and molar teeth on the right side. His temperature is 38.7EC (101.6EF),
orally. He has bloody, thick, green mucus coming from his right nostril. The remainder of his physical examination is normal. You suspect
maxillary sinusitis on the basis of the clinical findings.

27. Before beginning antibiotic treatment in this patient, it is necessary to first do which of the following?

(A) Confirm the diagnosis with CT films of the sinus


(B) Confirm the diagnosis with plain x-ray films of the sinus
(C) Confirm the diagnosis with transillumination of the sinus
(D) Request consultation with a dentist
(E) No additional steps are necessary

28. The appropriate steps are taken. In prescribing antibiotic therapy for this patient, it is most important to remember which of the
following?

(A) Antibiotics are ineffective unless there is a concomitant surgical drainage procedure
(B) He is likely to be allergic to penicillin
(C) He needs to take medication for more than 7 days
(D) Only bactericidal antibiotics are effective
(E) Ordinarily at least two antibiotics are given

He is treated and initially feels much better. However, soon thereafter he developed a headache, right ear pain and painful stiff neck; he
spikes a temperature to 39.3EC (102.7EF), orally. Extraocular movements are normal.

29. Which of the following is the most likely explanation for these new symptoms?

(A) Allergic reaction to the antibiotic(s)


(B) Associated meningeal inflammation or infection
(C) Development of cavernous sinus thrombosis
(D) Direct spread of infection from the maxillary to the mastoid sinus
(E) Obstruction of the orifice of the maxillary sinus with a mucous plug

END OF SET

48
30. A 50-year-old African-American man with severe chronic obstructive pulmonary disease returns to the office following a recent
evaluation for possible lung transplantation in another city. He says he had been considered a suitable candidate, in all respects, but
was rejected by the transplant program when a random urine test was positive for a nicotine metabolite. He had previously told you
that he had stopped smoking 3 years ago. He stands by this and is at a loss to explain the positive urine test. He wants to know what
he should do now. Which of the following is the most appropriate next step?

(A) Advise him again to stop smoking and refer him to another transplant program
(B) Advise him that the transplant program cannot turn him down on this basis, according to the Americans with
Disabilities Act
(C) Contact the transplant program to learn their reasons for turning him down
(D) Explain to the patient that transplantation is out of the question as a result of what has occurred
(E) Write to the transplant program and insist that they give him another opportunity

Items 31-33

A 43-year-old teacher, who is the mother of three children, comes to the office for evaluation of high blood pressure. An elevated blood
pressure was first detected 1 week ago during a routine screening at health fair in a local shopping mall. On examination today her blood
pressure is 145/95 mm Hg.

31. Which of the following is the most appropriate next step?

(A) Advise her to monitor her blood pressure twice a day at home and return to the office in 6 months
(B) Ask her to return for reexamination after her next menstrual cycle
(C) Ask her to return for reexamination in 2 weeks
(D) Ask her to return for reexamination in 4 months
(E) Measure her blood pressure after she exercises for 5 minutes

32. Hypertension is confirmed. She is started on a low-sodium diet and an antihypertensive medication. In addition, she should be
advised that she should do which of the following?

(A) Maintain ideal body weight


(B) Restrict her physical activities
(C) Seek less stressful employment
(D) Take 300 mg of aspirin, daily
(E) Use a combination oral contraceptive

33. On physical examination 1 year later she is asymptomatic and her blood pressure is 140/85 mm Hg. She is continuing to take the
prescribed antihypertensive medication. Which of the following is the most appropriate management at this time?

(A) Continue the antihypertensive therapy


(B) Discontinue the antihypertensive therapy
(C) Order a chest x-ray film and electrocardiography
(D) Order complete blood count, serum electrolyte concentrations and liver chemistry profile
(E) Reduce the dosage of the antihypertensive medication by 50%

END OF SET

49
34. A 24-year-old woman comes to the office because of concerns about sexual function. The patient recently has remarried after being
divorced from an abusive partner. She is currently unable to complete intercourse with her new husband due to intense vaginal pain
on attempts at penetration. She was able to have intercourse successfully early in her first marriage. She is orgasmic with other
stimulation but cannot tolerate digital or other penetration. Vital signs are: temperature 37.0EC (98.6EF), pulse 72/min, respirations
18/min and blood pressure 118/64 mm Hg. Physical examination is normal; however, the patient is unable to tolerate the speculum
examination because of pain. Which of the following is the most likely mechanism for this condition?

(A) Change in vaginal flora because of a new partner


(B) Conversion disorder
(C) Inadequate lubrication
(D) Inflammation of vestibular glands
(E) Vaginal muscle spasm

35. A 67-year-old woman who is a regular patient calls the office because she has developed severe muscle weakness, muscle cramps
and polyuria. She began treatment 6 weeks ago with 50 mg of chlorthalidone daily for mild-to-moderate essential hypertension. The
most likely explanation for her symptoms is the development of which of the following?

(A) Diabetes mellitus


(B) Hypokalemia
(C) Hypomagnesemia
(D) Hyponatremia
(E) Metabolic acidosis

36. A recent, large clinical trial assessed the effect of digoxin on morbidity and mortality in patients with chronic congestive heart
failure (CHF). In the trial, more than 7000 patients with left ventricular ejection fractions of 0.45 or less were randomly assigned to
receive digoxin or placebo. All patients also were treated with diuretics and an angiotensin-converting enzyme (ACE) inhibitor. The
patients were observed for an average of 37 months. During the clinical trial, 34.8% of patients treated with digoxin and 35.1% of
patients treated with placebo died (relative risk=0.99; confidence interval 95%=0.91 to 1.07, p=0.80). The best interpretation of
these data is that in combination with a diuretic and ACE inhibitor, digoxin therapy results in which of the following?

(A) A beneficial effect on mortality rates


(B) A beneficial effect on patients with CHF
(C) No conclusive evidence of effect because of the limited power of the study
(D) No effect on CHF
(E) No effect on mortality rates

37. A 6-year-old Mexican-American boy is brought to the office by his mother because of the gradual onset of abdominal pain. The visit
was prompted by the child's first grade teacher who sent him home from school because he appeared ill. The boy has been above the
recommended weight for his height at every visit since he became your patient 5 years ago. On physical examination today the child
does not appear to be in acute distress. Abdominal examination discloses normal bowel sounds and no tenderness, and is otherwise
inconclusive. Which of the following is the most appropriate next step?

(A) Ask the mother about problems separately from her son
(B) Call the school to check on details of the onset of pain
(C) Do rectal examination
(D) Obtain serum liver chemistry profile
(E) Order supine x-ray film of the abdomen

50
38. A 44-year-old general surgeon is convicted of narcotic abuse. He successfully completes a 3-month drug rehabilitation program.
Following this treatment, his privilege to practice medicine will be determined by which of the following?

(A) American Board of Surgery


(B) Credentials committee of his hospital
(C) State medical board
(D) US Drug Enforcement Agency
(E) He will not be permitted to practice

39. A 76-year-old woman returns to the office because of aching and weakness in her arms to the point where she cannot lift her arm to
brush her hair. Physical examination shows no muscle tenderness or other evidence of joint disease in both arms. The aching
improves when she takes the prescribed nonsteroidal anti-inflammatory drug (NSAID). She also describes tenderness over the right
parietal area of her scalp. Physical examination of the scalp shows no lesions. Which of the following is the most appropriate next
step?

(A) Increase the dose of the NSAID


(B) Order determination of erythrocyte sedimentation rate
(C) Order determination of serum rheumatoid factor
(D) Order x-ray films of the cervical spine
(E) Refer her for psychiatric counseling

40. A 34-year-old laborer comes to the office because of a 2-kg (5-lb) weight loss and an increased appetite. He has diabetes mellitus
and has been taking insulin in divided doses. He says that home monitoring of his serum glucose concentration has shown values
from 280 mg/dL to 320 mg/dL. Which of the following is the most appropriate management?

(A) Add metformin


(B) Change to another type of insulin
(C) Increase his caloric intake
(D) Increase his insulin dose
(E) Redistribute his caloric intake

41. An 84-year-old woman is brought to the office by her daughter, who is your patient. The mother has just moved in with the family
because she can no longer care for herself due to progressive, long-standing dementia. The daughter hopes you will help take care of
her mother. On physical examination the mother does not respond to your words or to the physical examination. You notice that she
smells of urine. On examination of the pelvis there is a diffuse erythematous rash extending over the perineum and the medial
thighs bilaterally. You suspect the rash relates to urinary incontinence. The daughter is present at the mother's examination. Which
of the following is the most appropriate statement to the daughter?

(A) "How long have you left your mother in this condition?"
(B) "His is a rash caused by urine. My nurse will insert a Foley catheter."
(C) "This rash should respond to cleansing with mild soap and drying with a clean towel three times a day."
(D) "You should take better care of your mother."
(E) "Your mother needs to wear diapers."

51
42. You are discussing treatment options with a 75-year-old woman with chronic renal failure. Her medical history also includes
diabetes mellitus, hypertension, hypercholesterolemia, and a left above-the-knee amputation secondary to severe peripheral vascular
disease. She lives with her dog in a one-bedroom apartment in the suburbs and is dependent upon a wheelchair for ambulation. She
has a fifth-grade education. Her support system includes a home health aide, several friends who live 5 to 6 miles from her
apartment and Meals-on-Wheels. Which of the following is the most appropriate advice to give this patient regarding long-term
management?

(A) Dialysis is not an appropriate option because of her age and medical disabilities
(B) Home dialysis would permit the greatest independence and convenience
(C) Peritoneal dialysis is not an option because of her diabetes mellitus
(D) She must name a health care surrogate to make decisions regarding her care
(E) You will arrange for dialysis at a center near her home if she can arrange transportation

Items 43-44

A 34-year-old, white, married business executive who is a patient in your practice comes to the office because of low back pain. After you
enter the examining room and establish rapport, he appears embarrassed and says that back pain is not why he is here. Upon further
questioning, he admits to a sexual encounter on a recent business trip and says he now has a greenish urethral discharge. Tests for
gonorrhea are positive. He admits to having exposed his wife since returning from the trip. He does not want his wife told about this.

43. Because his wife is also a patient in your practice, which of the following is the most ethical approach?

(A) Report the case to the state health department and let them handle contacting his wife
(B) Request that he inform his wife of her exposure and the need for treatment and offer to assist as necessary
(C) Suggest that he bring his wife to the office tomorrow so that you can explain the situation and the need for treatment
for both of them
(D) Telephone his wife after he leaves, inform her of the problem and request that she come to the office for treatment
(E) Treat him now and treat his wife when she comes to the office for her next annual Pap smear 2 weeks from now

44. The patient decides he will personally tell his wife that night. He calls you the next day stating that his wife is angry and distraught,
and she is contemplating divorce. She is so upset she cannot even come to the office for treatment. He wants to know what to do
now. Which of the following is the most appropriate next step?

(A) Call the wife to ask her to come to the office to be treated and to talk about her reaction to this situation
(B) Have a member of your nursing staff call the wife to stress the importance of coming to the office for treatment
(C) Make a house call to treat her and assess the situation
(D) Refer them to a marriage counselor and ask her to come to the office next week for treatment
(E) Refer them to a psychiatrist skilled in crisis intervention and give her an antidepressant medication until the next
available appointment in 3 weeks

END OF SET

52
45. A 28-year-old man comes to the office because he is concerned about his relationship with his 9-year-old son. He is a single father
who has full custody of his son. The father says that they always have been very close, but recently he has not had much energy to
spend quality time with his son. He does not understand why he has had this change in behavior. He says he feels well, and to his
knowledge, is healthy. He says that he works daily in a job he likes and has had no occupational problems. History and physical
examination lead you to suspect substance abuse. Abuse of which of the following substances would most likely explain his
behavior?

(A) Amphetamine
(B) Cocaine
(C) Marijuana
(D) Phencyclidine
(E) Psilocybin

46. A 28-year-old woman who is 20 weeks pregnant with her third child comes to the office for a routine prenatal visit. She has two
healthy children, ages 7 and 5 years. Her previous pregnancies were uncomplicated except for a cesarean delivery of the first child.
She began using cocaine soon after the birth of her second child, and she has been in and out of drug treatment programs for the
past 3 years. She has used crack cocaine on and off throughout this pregnancy. You have encouraged her to seek help; however, she
has made no attempt to abstain from using cocaine and she refuses to commit herself to another drug treatment program. In your
attempt to persuade this patient to stop using cocaine, you advise her that if she continues to use cocaine during the pregnancy, she
increases her risk for which of the following?

(A) Chorioamnionitis
(B) Gestational diabetes
(C) Placental abruption
(D) Placenta previa
(E) Preeclampsia

47. A 22-year-old college student calls your evening answering service because of an itchy, spreading rash over her right wrist and
lower arm for the past 2 days. You return her call, and she reports that she just returned from a weekend camping trip in the
mountains. She denies any fever, chest pain, dyspnea or red streaks up her arm. She states, "I'm feeling miserable and the itch is
killing me. I don't know if I was bitten by some insect or a tick. What should I do?" In counseling the patient over the telephone,
which of the following is the most appropriate response?

(A) "From what you describe I think you should go directly to a dermatologist. Please come to my office in the morning for
a referral."
(B) "Go to your pharmacist and ask for an over-the-counter antihistamine and Burow's solution, which you should apply
with wet compresses, and please come to my office in the morning."
(C) "I will call in antibiotics to the pharmacy so that you can begin treatment tonight. I would like to see you in my office
tomorrow."
(D) "Unfortunately I cannot prescribe any treatment without first examining you. Please call my office in the morning for
an appointment."
(E) "You should go to the nearest emergency department immediately for treatment and come to my office tomorrow
morning for follow-up."

48. During a sports physical examination, a 13-year-old boy expresses concern about his height. He was in the 10th percentile for
height throughout childhood and now is just above the 5th percentile. His sexual maturation rating is Tanner stage 2 for both
genitalia and pubic hair. He should be told which of the following?

(A) He is near his final height


(B) He probably has a slight deficiency of growth hormone
(C) He should increase his daily caloric intake
(D) He will grow several more inches
(E) He will need testosterone injections for several months

NOTE: THIS IS THE END OF THE OFFICE/HEALTH CENTER BLOCK.


ANY REMAINING TIME MAY BE USED TO CHECK ITEMS IN THIS BLOCK.

53
Block 2: Office/Health Center
Items 49-96; Time - 60 minutes

You see patients in two locations: your office suite, which is adjacent to a hospital, and at a community-based health center.
Your office practice is in a primary care generalist group. Patients are seen for routine and urgent care at the office and
health center. Most of the patients you see are from your own practice, although occasionally you will see a patient cared for
by one of your associates and reference may be made to the patient's medical records. Known patients may be managed by
telephone, and you may have to respond to questions about information appearing in the public media, which will require
interpretation of the medical literature. The laboratory and radiology departments have a full range of services available.

ALL ITEMS REQUIRE SELECTION OF ONE BEST CHOICE.

49. A 14-year-old boy is brought to the walk-in clinic by his father late on Saturday afternoon because his left ear is swollen and
painful. The boy's ear has been black and blue since he injured it in a wrestling match 3 days ago. Symptoms have increased
significantly following a repeat injury 3 hours ago. On physical examination, his left ear is markedly swollen and tender to
palpation. Which of the following is the most appropriate next step?

(A) Reassure him and start aspirin therapy


(B) Reassure him and start codeine therapy
(C) Recommend that he apply cold packs to the ear for the next 12 hours
(D) Recommend that he apply hot packs to the ear for the next 12 hours
(E) Refer him to a surgeon for immediate drainage of the lesion

Items 50-51

A 6-week-old infant is brought to the health center by his mother because of projectile vomiting. You have not seen this infant before
and the mother says he has not been seen by a physician since birth. Weight at birth was 3550 g (7 lb 13 oz) and physical
examination was normal. Examination today is normal except for weight, now 4000 g (8 lb 13 oz), and slight dehydration.

50. The best management for this infant is to

(A) elevate the head of his crib to relieve gastroesophageal reflux


(B) order supine and decubitus x-ray films of the abdomen
(C) order ultrasonography of the pylorus
(D) order an upper gastrointestinal barium study
(E) place him on a clear pediatric electrolyte solution

51. The appropriate diagnostic test is done and is equivocal. The imaging modality necessary to help diagnose this infant's illness
and that is most operator-dependent is

(A) CT scan
(B) radionuclide gastric emptying time studies
(C) routine x-ray films
(D) ultrasonography
(E) upper and lower gastrointestinal barium studies

END OF SET

54
52. A 75-year-old woman comes to the clinic because she has band-like burning pain in the right upper quadrant extending from
the epigastrium around to the midline of the back. On physical examination, there is no abdominal tenderness. Findings on
ultrasonography of the gallbladder are normal; serum amylase concentration is normal. The most likely diagnosis is

(A) acalculous cholecystitis


(B) chronic relapsing pancreatitis
(C) diverticulitis of the cecum
(D) herpes zoster
(E) penetrating duodenal ulcer

53. A 2-year-old boy who has recently become one of your patients is brought to the clinic by his mother for a follow-up visit of a
chromosome analysis done 1 month ago. This child has minor dysmorphic features, and growth and developmental delay.
Chromosome analysis showed a small unbalanced chromosome translocation, with extra chromosomal material at the tip of
chromosome 3. The cytogenetics laboratory requested blood samples from both parents for follow-up studies. The parents are
divorced, and the mother has custody of the child. The relationship between the parents is hostile. The mother has been tested
and has normal chromosomes without evidence of translocation. At today's visit, she reacts angrily when the issue of
contacting the child's father for testing is raised. She states that he abandoned them, and that he has no interest in his child.
She refuses to cooperate in contacting the father, who could be a translocation carrier. You do not know the father, but an
office worker told you that he lives in a nearby town. The mother says that he is living with a new girlfriend. The most
appropriate next step should be to

(A) attempt to identify the father's physician and work with that physician to obtain chromosome studies on the
father
(B) contact the father by telephone and arrange for him to give a blood sample at a local hospital
(C) document your attempts to work with the mother but proceed no further, since you have no physician-patient
relationship with the father
(D) help the mother deal with her anger and educate her regarding the potential benefit to her son and others if the
father's chromosome studies are done
(E) send the father a letter (expressing few details about the patient) and suggest that he contact your office for an
appointment and further discussion of his child

54. An 18-month-old white infant is brought to the clinic because of pallor and irritability. Her mother says the infant's diet
consists almost exclusively of whole milk, approximately 40 oz per day. On physical examination, the infant has a pulse of
160/min, respirations of 50/min, and normal heart sounds with a grade 2/6 systolic ejection murmur. Liver is palpable 3 cm
below the right costal margin. Laboratory studies show:

Blood
Hematocrit 13%
Hemoglobin 3 g/dL
Mean corpuscular volume 48 µm3
Platelet count 400,000/mm3
Reticulocyte count 0.8% (N=0.5-1.5% of red cells)
WBC 12,000/mm3

Following appropriate evaluation, which of the following is the most appropriate treatment?

(A) Administration of oral folate therapy


(B) Administration of parenteral iron therapy
(C) Administration of parenteral vitamin B12 (cyanocobalamin) therapy
(D) Transfusion with packed erythrocytes
(E) Transfusion with whole blood

55
Items 55-56

A 75-year-old white woman returns to the office after 6 months of missed appointments. She says she is feeling depressed. You
have been treating her for years for a variety of disorders, including bipolar disorder, hypothyroidism, atrial fibrillation, peptic ulcer
disease and hypertension. She takes daily lithium, levothyroxine, haloperidol, sertraline, benztropine, digoxin, propranolol,
ranitidine and warfarin. At this time she says, "I'm doing fine except for shakiness in my hands." Vital signs are pulse 78/min with
an irregularly irregular rhythm, and blood pressure 160/95 mm Hg. Physical examination shows she has fine tremor of the hands
when they are extended. She says her mood is "a little depressed," and she has no hallucinations or delusions. On memory testing,
she recalls one of three objects after 2 minutes.

55. Which of the following is the most likely cause of the patient's depression?

(A) Benztropine
(B) Digoxin
(C) Haloperidol
(D) Propranolol
(E) Ranitidine

56. You decide to prescribe hydrochlorothiazide. Of her current medications, the hydrochlorothiazide would most likely cause a
serious drug interaction with which of the following?

(A) Haloperidol
(B) Lithium
(C) Ranitidine
(D) Sertraline
(E) Warfarin

END OF SET

57. A 42-year-old woman with a history of multiple sclerosis comes to the office because she had a sudden loss of vision in the
right eye. She has no history of diplopia. On examination, external ocular movements are normal but funduscopic
examination shows pallor of the optic disk. This patient's condition is most likely due to demyelination of which of the
following?

(A) Medial longitudinal fasciculus


(B) Oculomotor nerve
(C) Optic nerve
(D) Trigeminal nerve
(E) Visual cortex

56
58. A 44-year-old Irish-American woman has had arthritis for 10 years, for which she has seen many physicians. She has used
many medications and devices, including copper bracelets from Mexico given to her by friends. She seeks your help because
for the past several months she has had increasing pain and stiffness in her hands. Her hands now show moderate ulnar
deviation of the fingers and she says her wrists and knees also hurt. She has had increasing fatigue for about 1 month, along
with a weight loss of 1.8 to 2.2 kg (4 to 5 lb). Review of her medical records, which she has brought with her, convinces you
that the initial diagnosis of rheumatoid arthritis is correct. She says, "I had several drop attacks during the past 3 months."
She characterizes these attacks as episodes of weakness and loss of feeling in her legs for several minutes. During one of
these episodes, she became incontinent. She currently takes aspirin about four times a day and an occasional dose of
ibuprofen. On physical examination she has facial plethora and swollen and painful metacarpophalangeal and knee joints,
bilaterally. The rest of the examination is normal. Which of the following is the most likely cause of her "drop attacks?"

(A) Adrenal insufficiency


(B) Anxiety
(C) Atlanto-axial instability
(D) Cardiac arrhythmia
(E) Cerebral ischemia

59. A 3-year-old boy is brought to the office by his father because of a 3-month history of decreased activity, poor appetite,
sporadic vomiting, clumsiness and speech regression. Since his birth his family has lived in an old area of the city where
there is demolition of old buildings. Examination of a peripheral blood smear is likely to show which of the following?

(A) Basophilic stippling of erythrocytes


(B) Degranulation of eosinophils
(C) Diminished numbers of platelets
(D) Howell-Jolly bodies
(E) Macrocytic erythrocytes

60. A 68-year-old woman comes to the office for flexible sigmoidoscopy as part of a yearly screening. A 3-cm polyp is found in
the sigmoid colon and is removed. She returns now to the office, 6 hours later, complaining of left lower quadrant pain,
fever, nausea and vomiting. Vital signs are: temperature 38.1°C (100.6°F), pulse 110/min, respirations 26/min and blood
pressure 120/60 mm Hg. Abdominal examination discloses bowel sounds, tenderness and guarding in the left lower
quadrant. Rectal examination shows no stool and only tenderness superiorly. The most appropriate next step is to

(A) obtain an angiogram to rule out intestinal ischemia


(B) obtain immediate consultation with a surgeon
(C) pass a soft rubber rectal tube under fluoroscopy
(D) repeat the flexible sigmoidoscopy in order to evaluate the operative site
(E) start hydrocortisone, intravenously, to decrease any inflammatory response

61. A 32-year-old man and his 29-year-old wife are being evaluated for infertility. The woman's gynecologist reports that the
woman had a normal anatomic and physiologic evaluation and suggests the need for assessment of potential male factors.
On examination, the man is 188 cm (6 ft 3 in) tall with fair skin and little facial hair. His testicles are small and firm, and he
has mild gynecomastia. No sperm are seen on analysis of his semen. Which of the following tests is most likely to establish
the underlying cause of the infertility?

(A) Karyotype from peripheral leukocytes


(B) Serum estrogen and testosterone concentrations
(C) Serum gonadotropin concentrations (follicle-stimulating hormone and luteinizing hormone)
(D) Serum prolactin concentration
(E) Testicular ultrasonography

57
Items 62-63

A 4-day-old Greek-American neonate is brought to the office because of the development of yellow skin and a rash 1 day after
hospital discharge. She weighed 3400 g (7 lb 8 oz) at birth and is the product of a normal pregnancy. The mother is now gravida 2,
para 2 and she is blood type A, Rh positive. The neonate is blood type O, Rh positive with a negative direct Coombs test. She had
an Apgar score of 8 and 9 at 1 and 5 minutes respectively. The neonate was breast-feeding and was doing well at the time of
discharge. Yesterday, the mother says, the neonate developed about 20 small red spots over her face, trunk and extremities. Today
on physical examination there are many papules that have small vesicles with clear to slightly turbid fluid. Her skin color has
become yellow. The mother says that she continues to feed well. On physical examination the neonate weighs 3250 g (7 lb 2 oz).
There is scleral and skin icterus. No organomegaly or adenopathy is noted. Studies on the neonate show a serum total bilirubin
concentration of 8.7 mg/dL, and concentration of serum conjugated (direct) bilirubin is 0.7 mg/dL.

62. In addition to scheduling a follow-up visit in 1 week, the most appropriate advice to give the mother regarding the icterus is
to

(A) ask her to avoid eating foods containing large quantities of carotene
(B) begin administering small doses of phenobarbital to the neonate
(C) discontinue breast-feeding until the jaundice has disappeared
(D) recommend home phototherapy for the neonate
(E) recommend no change in child care or feeding of the neonate

63. The appropriate steps are taken. You explain to the mother that the neonate's rash is probably erythema toxicum and you
prescribe

(A) daily wet-to-dry povidone-iodine (Betadine®) soaks with 1×1 gauze pads on each vesicle
(B) polymyxin ointment applied twice a day
(C) routine skin care with soap and water
(D) scrubbing with entsufon cleanser each day firmly enough to unroof the vesicles
(E) 0.5% hydrocortisone cream applied twice a day

END OF SET

58
Items 64-65

A 10-year-old girl, who has been undergoing treatment for chronic juvenile rheumatoid arthritis for the past 3 years, is brought to
the office because of painful swelling of the right knee. She has had three episodes of painless swelling of her left knee and ankle,
which have subsided spontaneously with rest and aspirin therapy. She has used no medications between episodes. On physical
examination today there is pronounced redness and warmth around the right knee, and a large effusion is present. Attempts at
active and passive motion cause severe pain.

64. Which of the following is the most appropriate step at this time?
99m
(A) technetium bone scan
(B) Joint aspiration
(C) Serum antinuclear antibody titer
(D) Serum rheumatoid factor assay
(E) X-ray films of the joint

65. Which of the following new symptoms or findings, if present, would best indicate the need for further diagnostic studies?

(A) Decreased viscosity of joint fluid


(B) Diffuse increase in 99mtechnetium uptake around the knee on bone scan
(C) Positive Gram stain of joint fluid
(D) Positive serum rheumatoid factor test
(E) Soft-tissue swelling seen on x-ray films

END OF SET

Items 66-67

A 38-year-old homemaker and mother of four children (ages 5 to 12 years) has been coming to you for management of tension
headaches that have not improved with trials of several appropriate medications. She has been married to a police officer for the
past 6 years. You ask if she has been under extra stress, and she begins to cry. You notice bruises on her arms. On further
questioning, you learn that her husband hits her whenever he is drunk, which is at least 2 nights per week. She says, "He is nice...a
good husband when he's sober. But when he drinks, oh he's awful! He accuses me of cheating on him. Last night he said he would
kill me if I try to leave." Her husband is also a patient of yours.

66. Which of the following is the most appropriate intervention?

(A) Advise her to leave her home with her children and move in with her relatives
(B) Contact her husband's supervisor to discuss recent stress levels on the job
(C) Gather more information while remaining neutral, since both the husband and wife are your patients
(D) Refer her to a domestic violence program
(E) Seek a restraining order against her husband on her behalf

67. Which of the following is the most important question to ask at this time?

(A) "Do you think this might be causing your headaches?"


(B) "Has your husband also lost his temper with any of the children?"
(C) "Have you been drinking at the time of the fights?"
(D) "Have you or your husband been receiving any kind of counseling?"
(E) "Why have you stayed in this marriage?"
END OF SET

59
68. A 24-year-old woman comes to the office for a gynecologic examination. This is her first visit and she has no complaints.
She tells you that she has not had a Pap smear for several years. Menarche was at age 12 years and she has had normal
menstrual cycles since then. She has had several sexual partners in the past but has been with her current partner in a
monogamous relationship for 1 year. She reports that she had a chlamydial infection that was treated several years ago, but
she denies a history of other sexually transmitted diseases. She has never been pregnant. On physical examination her cervix
appears friable with a slight area of ulceration. There are several perineal and vaginal lesions that appear as small
"cauliflower-like" projections. The results of the Pap smear, which return in 1 week, show mild dysplasia (LGSIL). Which of
the following factors in this patient's history most closely correlates with the abnormal finding on Pap smear?

(A) Condylomata acuminata


(B) Condylomata lata
(C) Early age at menarche
(D) History of chlamydia
(E) Nulliparity

69. A 50-year-old woman comes to the office for the first time because of recurrent abdominal pain. Review of her extensive
medical chart, which she has brought with her, discloses that she has a long history of varying physical complaints.
Definitive causes for these complaints have not been found despite extensive diagnostic studies, consultations with many
physicians and several surgical explorations. She gives dramatic and exaggerated descriptions of her present and past
symptoms, and she makes conflicting statements about her history. She has been hospitalized at least 23 times since age
18 years. Which of the following is the most likely diagnosis?

(A) Borderline personality disorder


(B) Conversion disorder
(C) Histrionic personality disorder
(D) Occult medical disorder
(E) Somatization disorder

70. At a routine physical examination, the Pap smear of a 27-year-old woman shows evidence of marked inflammation
suggestive of moderate dysplasia (HGSIL). Her last Pap smear 2 years ago was normal. Pelvic examination today is normal.
She has never been pregnant and her menstrual periods are regular. She has been in a stable relationship with the same man
for 3 years and she uses a diaphragm with spermicidal jelly for contraception. The best next step is to

(A) advise the patient that her partner should use condoms for contraception and repeat the Pap smear in 3 months
(B) do colposcopic examination of the cervix after application of 5% acetic acid solution
(C) do conization of the cervix
(D) reassure the patient and repeat the Pap smear in 3 months
(E) treat the patient with metronidazole for 2 weeks and repeat the Pap smear in 3 months

71. A 66-year-old African-American man who has been a patient for several years calls the office to report an episode of
apparently bloody urine. He is instructed to come to the office, where urinalysis confirms gross hematuria without
proteinuria or casts. The patient denies any pain and is anxious for an explanation. Physical examination is normal. The
most appropriate next step is to

(A) do a transrectal prostatic biopsy


(B) prescribe a 1-month course of trimethoprim-sulfamethoxazole
(C) schedule bilateral renal angiography
(D) schedule cystoscopy
(E) schedule infusion of the renal pelvis with silver nitrate

60
72. A 19-year-old white woman returns to the office 2 months after having a medroxyprogesterone injection for contraception.
She is complaining of nonstop bleeding since her menses 3 weeks ago. She is using eight pads a day. She denies any sexual
activity since she received the injection. She realizes that spotting is a side effect; however, she is anxious about the length of
time and the amount of the bleeding. She tells you, "I can't stand this, Doctor. I want the bleeding to stop now!" Repeat
pregnancy test is negative. Which of the following is the most appropriate treatment option for her bleeding?

(A) Conjugated estrogen therapy for 2 weeks


(B) Dilatation and curettage
(C) Reassurance and counseling
(D) A second injection of medroxyprogesterone
(E) Triphasic oral contraceptive therapy for one cycle

73. A 4-year-old boy is brought to the office because he has become unmanageable at his day-care center. At previous visits he
exhibited some behavior problems to which his mother did not set limits. He constantly interrupted situations, seeking his
mother's attention. She now reports that during the past few months his fighting, refusal to obey the day-care workers and
violations of "time out" have become much worse. He began to attend day-care at 6 weeks of age so that his mother could
return to work. His father works as a house painter and he is alcohol-dependent. The boy has a 6-month-old sister who also
attends the same day-care center. Records show his height and weight are at the 5th percentile, and his growth velocity is
normal. There were no complications during the pregnancy with this child and he has not had any significant medical
problems. His physical examination today is normal. The most likely cause for this child's worsening behavior is

(A) aggressiveness to compensate for a poor self-image caused by short stature


(B) attention-deficit/hyperactivity disorder
(C) a reaction to his father's drinking
(D) reduction in his mother's attention because of his new sibling
(E) a toxic reaction to organic fumes from his father's clothes and work materials

74. A 28-year-old woman of Scandinavian descent comes to the office because of fatigue, weakness and palpitations. She is
divorced and lives with her 4-year-old daughter. Complete evaluation shows that this patient has hyperthyroidism and mild
ophthalmopathy caused by Graves disease. Before initiating therapy, the patient wants to know what she can expect in the
future. In advising her about the prognosis, the most accurate statement is:

(A) Graves ophthalmopathy will resolve as thyroid hormone secretion is lowered


(B) Malignant degeneration of the thyroid is a common complication
(C) She will not be able to become pregnant
(D) The thyroid will continue to increase in size with any nonsurgical treatment
(E) Untreated patients are at increased risk for cardiac arrhythmias

61
75.

A 58-year-old man comes to the office because of a lesion on the lip, which is shown. The patient says he has had the lesion
for about 9 months. He has not seen a physician for 5 years and he is in the office today only because, he says, "My wife
made me come." On physical examination the lower lip is fixed to the anterior aspect of the mandible. The most likely
diagnosis is

(A) basal cell carcinoma


(B) keratoacanthoma
(C) leukoplakia
(D) melanoma
(E) squamous cell carcinoma

62
76. An 11-year-old girl is brought to the office because of pain in her left calf that she first noted 4 weeks ago and that has
gradually increased. Before the onset of symptoms she was running 10 to 12 miles per week; now she is unable to run
because of the leg pain. There is no history of acute injury despite her intensive training schedule. She is a sprinter who has
won a local qualifying event for a national competition. There is tenderness of palpation over the proximal portion of the
posterior calf musculature. X-ray films are shown. The most likely diagnosis is

(A) benign neoplasm of bone


(B) malignant neoplasm of bone
(C) metabolic bone disease
(D) osteomyelitis
(E) stress fracture

63
77. A 68-year-old man with documented alcohol abuse returns to the office because of abdominal pain and bloating. When you
meet with him, he appears dejected and his eye contact is poor. Physical examination is normal. Since his last visit, he has
moved from the neighborhood where he had lived for 40 years. In addition, he mentions that approximately 6 months ago
breast cancer was diagnosed in his wife; she is currently receiving radiation therapy for bony metastases. The most important
next step in management of his symptoms is evaluation for

(A) delirium tremens


(B) gastrointestinal bleeding
(C) pancreatic carcinoma
(D) situational anxiety disorder
(E) suicidal ideation

78. A 25-year-old woman who is pregnant with her third child comes to the office for a regular prenatal visit. Medical history
shows that she developed deep vein thrombosis of the left calf in the 22nd week of her last pregnancy 2 years ago. She is now
26 weeks pregnant, and she complains of left calf tenderness during the examination. Deep vein thrombosis is confirmed by
Doppler ultrasonography. The most appropriate management is to

(A) administer intravenous heparin initially, followed by warfarin until delivery


(B) do venous ligation proximal to the point of obstruction
(C) prescribe heparin until delivery
(D) prescribe warfarin until delivery
(E) prescribe heparin and indomethacin therapy until delivery

79. A 69-year-old Chinese-American man with diabetes mellitus had a myocardial infarction 2 years ago. He has had
exertional angina since then and has been taking propranolol. During the past few days he has had one episode of chest pain
at rest, two episodes postprandially and one at night. Electrocardiogram reveals an old myocardial infarction. The most
appropriate treatment is to

(A) admit him immediately for cardiac monitoring and adjustment of therapy
(B) admit him immediately for coronary artery bypass surgery
(C) decrease the dosage of propranolol and adding nitrates and salicylates
(D) increase the dosage of propranolol and having him return in 1 week
(E) advise resting from work and sedation at night and digitalization

64
Items 80-82

You care for a family that consists of a 43-year-old husband, a 42-year-old wife, a 15-year-old daughter and a 12-year-old son.
Each family member is healthy. The 77-year-old maternal grandmother lived with the family until 4 weeks ago when she died
suddenly after a prolonged respiratory illness. Autopsy revealed that she had active pulmonary tuberculosis at the time of her death.
The organism tested sensitive to all commonly used anti-tuberculosis drugs.

80. In following up on the grandmother's illness, the most appropriate first step in managing this family is to

(A) obtain leukocyte count and erythrocyte sedimentation rate on all family members
(B) obtain sputum or gastric washings of all family members for culture for acid-fast bacilli
(C) place PPD skin tests on all family members
(D) place TB tine tests on the two children and PPD skin tests on the adults
(E) schedule bronchoscopy and alveolar lavage on the adults

81. Complete work-up of each family member reveals no evidence of tuberculosis. The most appropriate next step in
management is to prescribe

(A) isoniazid and rifampin for all family members


(B) isoniazid and rifampin for the adults only
(C) isoniazid for the children
(D) rifampin for the adults
(E) no medications for any family member

The father mentions that his mother-in-law spent many hours with a nephew in the weeks prior to her death. The nephew is 26
years old, has lymphoma and recently completed a course of chemotherapy. You agree to see the nephew, and as part of a thorough
examination, you place a PPD skin test.

82. The nephew's PPD skin test is positive. Complete evaluation of the nephew reveals no evidence of active tuberculosis. The
most appropriate pharmacotherapy at this time is

(A) isoniazid
(B) isoniazid and rifampin
(C) isoniazid, rifampin and ethambutol
(D) isoniazid, rifampin and streptomycin
(E) rifampin, ethambutol and pyrazinamide

END OF SET

65
Items 83-84

A 10-year-old white girl is brought to the office for her yearly physical examination. According to her mother the girl has been
teased regularly by other children because she is overweight. Both parents are obese. The girl’s growth chart is shown.

83. In reviewing the girl’s history you recognize that her weight places her at increased risk for

(A) Cushing syndrome


(B) delayed menarche
(C) hypothyroidism
(D) nocturnal enuresis
(E) slipped capital femoral epiphysis

84. In reviewing the management options for this patient, the most appropriate step is to

(A) give her a written diet to follow


(B) recommend a behavior-oriented treatment program
(C) refer her to a commercial weight-loss center for diet management
(D) refer her to an endocrinologist for hormonal assessment
(E) tell her not to worry because she will "grow into her weight"
END OF SET

66
Items 85-86

A 27-year-old man comes to the office because of a 1-week history of right knee pain. He says he jogs 3 miles a day and that the
pain in his knee worsens during his run. On physical examination his gait appears to be normal. Examination of the right knee
reveals tenderness and fullness over the medial collateral ligament.

85. On physical examination there is most likely to be

(A) increased anterior laxity with anterior pressure on the tibia


(B) increased posterior laxity with posterior pressure on the tibia
(C) pain on compression of the patella against the femur
(D) pain during internal and external rotation of the tibia while compressing it against the femur
(E) a painful clicking sensation with inward rotation of the foot and extension of the knee

86. The most accurate statement concerning this patient's condition is that he will need

(A) to choose a different type of activity for his exercise program


(B) initial treatment with rest, ice packs and isometric exercises
(C) referral for an orthotic device
(D) referral for arthroscopy
(E) treatment with knee immobilization and crutches

END OF SET

87. A 38-year-old white letter carrier returns to the office for follow-up of an abnormal liver chemistry profile ordered 3 weeks
ago during a routine examination. At that time, his physical examination was normal, but he had a serum AST
concentration of 72 U/L. His serum bilirubin and alkaline phosphatase concentrations were normal. History includes an
episode of hepatitis A at age 22 years. He has no history of transfusions or intravenous drug use. He drinks two to three beers
daily. Today's follow-up test results show:

Serum
Anti-HAV Positive
Anti-HBs Negative
HBsAg Positive
HBeAg Positive

Which of the following is the most appropriate next step?

(A) Begin interferon-alfa therapy


(B) Begin corticosteroid therapy
(C) Have him cease all alcohol consumption and retest him in 2 months
(D) Order hepatitis B virus DNA polymerase study
(E) Schedule liver biopsy

88. A 6-month-old Latino infant is brought to the office by his parents because of intermittent swelling in his right scrotum.
They say the swelling is more pronounced when he cries. The swelling has never been red or "stuck." A right inguinal
hernia is confirmed on physical examination. In discussing repair of the hernia with the parents, you should inform them
that:

(A) Herniorrhaphy can be postponed until age 2 years because many hernias close spontaneously
(B) Herniorrhaphy can be postponed until age 12 years because oligospermia does not develop before age 12
(C) Herniorrhaphy should be scheduled at the earliest convenient time
(D) Herniorrhaphy should be scheduled as an emergency
(E) There is no need to repair a hernia in childhood unless incarceration occurs

67
Items 89-90

A 35-year-old man who is a new city employee was driving a city-owned car when he was struck from behind by another car. He
experienced immediate neck and shoulder pain and was seen and examined by you in the emergency department. Physical
examination showed slight cervical muscle spasm. Cervical spine and shoulder x-ray films at that time were normal. You
reassured him that no structural damage was evident on the studies. He was treated with nonsteroidal anti-inflammatory drugs
(NSAIDs) and was scheduled for a return visit in the office in 2 weeks. At the return visit, he was slightly improved but said he had
been unable to return to work. Physical examination was normal. You prescribed physical therapy and scheduled a return visit in 4
weeks. Today, at the 4-week return visit, he insists he is not improved and demands everything be done to "find out why I still
hurt." Physical examination including neurologic examination, is normal. Repeat x-ray films of the cervical spine are normal.

89. At this time it is most appropriate to

(A) inform the patient that no additional studies are needed


(B) order a CT scan of the cervical spine
(C) order an MRI of the cervical spine
(D) order studies the patient requests
(E) refer him to a neurologist

90. The patient returns 1 month later with the complaint of persistent neck pain and of some "funny feeling" and pain in his
right hand. You learn his prior employment required repetitive use of both hands. He insists that he had no problem with his
hands and wrists prior to his accident. Neurologic examination at this time discloses only hypesthesia of the medial aspect of
his right little and ring fingers. The most appropriate next step is to

(A) apologize for not recognizing a structural problem from the outset
(B) discuss the possibility that he is malingering
(C) explain that this may be a problem unrelated to the accident
(D) order cervical myelography with CT scan of the cervical spine
(E) refer him to a neurosurgeon
END OF SET

91. An 80-year-old African-American woman is brought to the office for the first time by her son because she has signs of
mildly decreasing mental function. She is having increasing trouble reading, writing and watching television. She has mild,
stable angina pectoris and she had an uncomplicated myocardial infarction 8 years ago. Physical examination now is normal
except for corrected visual acuity of 20/200 O.U., which appears to be caused by cataracts. Mini-mental state test score is
29 out of 30. Which of the following is the most correct statement about this patient's condition?

(A) Her daily activities would probably improve if she had cataract extraction with lens implantation
(B) Her diminished mental status is a contraindication for a cataract operation
(C) Her history of cardiac disease and advanced age are contraindications for a cataract operation
(D) Her mental status should be reevaluated in 1 year
(E) You need more information to decide whether she would be helped by a cataract operation

68
92. An 8-month-old infant with trisomy 21 (Down syndrome) has a grade 2-3/6 systolic ejection murmur heard best at the left
sternal border, but it can be heard all over the precordium. S2 is split normally and is loud. She has had two episodes of
pneumonia in the past 2 months. Which of the following is the most appropriate next step?

(A) Do a PPD skin test


(B) Initiate an immunologic evaluation
(C) Order sweat chloride test
(D) Presume the murmur is functional and schedule follow-up visits
(E) Seek consultation with a cardiologist

Items 93-94

A 17-year-old boy is brought to the office by his mother who says that he has been increasingly withdrawn and preoccupied for
several weeks. He has been sleeping poorly and has refused to leave the house for the past week because he believes he is being
followed. When his father insisted he attend school this morning, the patient threatened him with a knife. On examination, the
patient is sweating, has dilated pupils and is obviously fearful.

93. Rapport with this patient can best be established by asking which of the following?

(A) "Are you hearing voices?"


(B) "Do you have a special mission to accomplish?"
(C) "Do you think you are being persecuted?"
(D) "How do you feel others are treating you?"
(E) "Who do you think is following you?"

94. You arrange to admit him to the hospital. Following admission, which of the following is the most appropriate immediate
intervention?

(A) Administer haloperidol


(B) Administer lorazepam, intramuscularly
(C) Arrange a conference with the family
(D) Order CT scan of the head
(E) Order toxicologic screening of the urine

END OF SET

95. A 5-year-old boy is brought to the office by his mother because of recurrence of bed-wetting at night. He has a 3-month-old
sister who is healthy. Physical examination is normal. Urinalysis shows:

Specific gravity 1.010


Glucose Negative
Protein Negative
Microscopic 0-1 WBC/hpf, 0 RBC/hpf

Which of the following is the most important information to share with his parents?

(A) This condition will cease if they reprimand him for deliberately wetting the bed
(B) The condition is self-limiting, and they should take care to lessen the emotional impact on their child
(C) This is a potentially serious problem and could represent chronic inflammation of the kidneys
(D) This may be a precursor of diabetes mellitus
(E) This signifies a serious underlying emotional disorder

69
96. A 38-year-old woman who is a single mother of two small children comes to the office saying that she feels "halfway tired all
the time." Her physical examination is normal. Toward the end of the visit she says anxiously, "My children just don't listen.
They make me so angry that I feel out of control sometimes." She pauses. "Yesterday my 7-year-old daughter talked back to
me and I slapped her in the face." Her eyes fill with tears. Which of the following is the most accurate statement concerning
this patient?

(A) She should be reported to child abuse authorities


(B) She was clearly a victim of child abuse herself
(C) She would benefit from antidepressant medication
(D) She would benefit from family counseling
(E) She would exhibit better self-control if she were married

NOTE: THIS IS THE END OF THE OFFICE/HEALTH CENTER BLOCK.


ANY REMAINING TIME MAY BE USED TO CHECK ITEMS IN THIS BLOCK.

70
Block 3: Inpatient Facilities
Items 97-120; Time - 30 minutes

You have general admitting privileges to the hospital, including to the children’s and women’s services. On occasion you
follow your own patients in the critical care unit. Postoperative patients are usually seen on the regular surgical ward unless
the recovery room is specified. You may also be called to see patients in the psychiatric unit. There is a short-stay unit where
you may see patients undergoing same-day operations or being held for observation. Also, you may visit patients in the
adjacent nursing home/extended care facility and the detoxification unit.

ALL ITEMS REQUIRE SELECTION OF ONE BEST CHOICE.

97. A 67-year-old man is admitted to the hospital because of fever, malaise and weight loss for the past 6 weeks. Night sweats
began 4 weeks ago. His medical records show that he has had a tonsillectomy, adenoidectomy, mitral valve
commissurotomy and sigmoid colon resection for diverticulitis. His temperature has been as high as 38.3°C (101.0°F) daily.
Today, vital signs are: temperature is 38.8°C (101.8°F), pulse 100/min and respirations 14/min. The patient appears
chronically ill but is in no acute distress. Other physical findings include a cotton-wool exudate in the right eye, crackling
rales at the lung bases and a moderate blowing grade 2/6 pansystolic murmur. There is moderate left lower quadrant
abdominal tenderness. Leukocyte count is 11,500/mm3 with normal differential and hematocrit is 35%. Urinalysis shows
4 WBC/hpf and 10 RBC/hpf. Chest and abdominal x-ray films are normal except for slight left ventricular enlargement. At
this which of the following is the most likely diagnosis?

(A) Bacterial endocarditis


(B) Diverticulitis
(C) Hodgkin disease
(D) Pyelonephritis
(E) Vivax malaria

98. A 60-year-old man is admitted to the hospital because of acute pancreatitis. Laboratory studies show:

Serum Blood
Amylase 1,000 U/L Hematocrit 42%
Calcium 8.4 mg/dL WBC 14,000/mm3
BUN 5 mg/dL

Results of serum liver chemistry profile are normal. After 48 hours of fluid therapy and observation, a poor prognosis would
be indicated by which of the following laboratory studies?

(A) Serum alanine aminotransferase (ALT) of 106 U/L


(B) Serum amylase of 2,000 U/L
(C) Serum bilirubin of 4.2 mg/dL
(D) Serum calcium of 6.6 mg/dL
(E) Serum glucose of 200 mg/dL

71
Items 99-100

A 25-year-old man has been on life support systems for the past 48 hours following blunt head trauma.

99. Which of the following is the most important criterion to declare the patient brain dead and to permit removal of life support
systems?

(A) Bedside EEG showing no electrical activity


(B) Decorticate and decerebrate posturing
(C) Failure to respond to electroconvulsive stimuli
(D) Glasgow coma score of 3 or less
(E) Unenhanced MRI of the brain showing dilated ventricles

100. Brain death cannot be established in this patient if there is the presence of which of the following?

(A) Carotid blood flow


(B) Cremasteric reflexes
(C) Elevated serum aminoglycoside concentrations
(D) Hypothermia
(E) Marked cerebral edema

END OF SET

Items 101-102

A 57-year-old man who manages his own accounting firm has a 5-year history of malignant melanoma that has been treated with
local excision and immunotherapy. He now is admitted to the hospital for evaluation of constant pain in his back and left hip and
an 11 kg (24 lb) weight loss. He and his wife of 35 years are worried that "the cancer may be back." Pelvic and abdominal CT scans
show multiple bony metastases. He tells you, "I just want to die. I can't bear this."

101. Which of the following is the most appropriate initial intervention?

(A) Adjust his analgesic regimen


(B) Arrange for him to be transferred to a psychiatric service
(C) Begin antidepressant medication
(D) Initiate hyperalimentation
(E) Refer him to a cancer patient support group

102. Which of the following symptoms would be most suggestive of a major depressive syndrome in this patient?

(A) Anorexia
(B) Expressions of discouragement
(C) Insomnia
(D) Low energy
(E) Withdrawal from family

END OF SET

72
Items 103-104
Patient Chart
Sex: Female
Current age: 22 years
Sociodemographic information: Married, African-American, graduate student
Medical history: First pregnancy; uncomplicated delivery of 4267-g (9-lb 7-oz) male infant. Fourth degree laceration of perineum
Family history: Noncontributory
Current medications/drug information: Prenatal vitamins
Today’s visit
Height: 168 cm (5 ft 6 in)
Weight: 67 kg (148 lb)
Vital signs:
Temperature 37.2EC (99.0EF) Respirations 18/min
Pulse 68/min Blood pressure 128/86 mm Hg
Physical examination: Uterus palpable to level of umbilicus; vaginal laceration appears without inflammation, swelling or drainage, with
sutures intact. Nurses report normal lochia
Laboratory studies: Hemoglobin on first day post-partum, 10.8 g/dL

The patient whose chart is shown, is being seen in the maternity ward 24 hours after vaginal delivery and repair of a fourth degree
perineal laceration. She is able to walk to the bathroom and to void without difficulty, but she has not had a bowel movement since
delivery. The patient tells you that she is concerned about her insurance company requirement that she stay in the hospital no
longer than 48 hours post partum. She is worried that she will not be ready to leave tomorrow since she is breast-feeding and wants
more help from the nurses. She also says she is worried about bowel function after the perineal repair and wants to stay until she is
sure it will be normal. She asks if you can extend her stay to 72 hours post partum if she is not ready to leave tomorrow.

103. Which of the following is the most appropriate response to her request?

(A) "I'm sure the insurance company will understand if you need another day, so you take whatever time you need
before you go home."
(B) "Let me decide whether or not it is too soon for you to leave the hospital."
(C) "Let's see how you feel tomorrow and we can discuss the most appropriate time to leave then."
(D) "You concentrate on getting better and leave the insurance company to me."
(E) "Unfortunately I have no control over the insurance company, so you had better plan on leaving tomorrow."

104. Which of the following would be the most important indication for extending her hospital stay beyond 48 hours post
partum?

(A) Abdominal distention and lack of appetite at 48 hours post partum


(B) Lack of bowel movement post partum
(C) Need for nursing assistance with breast-feeding
(D) Palpation of the uterus above the pubic symphysis for more than 48 hours post partum
(E) Persistence of lochia for more than 24 hours post partum

END OF SET

73
105. A 56-year-old Japanese-American woman is scheduled for an abdominal operation. She has hypothyroidism that is
controlled with thyroid replacement medication and will be unable to eat or drink for 4 days following the procedure. She is
concerned about receiving her thyroid medication. She should be advised of which of the following?

(A) Although she will have symptoms of hypothyroidism, she will not be given the medication
(B) Although she will not receive the medication, she will have no adverse effects
(C) The medication will be administered daily while the nasogastric tube is clamped
(D) The medication will be discontinued temporarily before the operation
(E) She will receive the medication intravenously

Items 106-108

A 76-year-old woman is admitted to the hospital following a fall at her home. On physical examination, she is dazed, has no
memory of her fall and is unable to respond to any questions about her health. Her skin turgor is poor, and there is bruising around
her left hip. Vital signs are: temperature 36.1EC (97.0EF), pulse 72/min and regular and blood pressure 140/85 mm Hg. X-ray films
of the pelvis taken upon arrival at the hospital show a fracture of the left femoral neck. She is accompanied by her daughter-in-law,
who knows only a portion of her mother-in-law's past medical history. The patient drinks socially, does not smoke cigarettes and
has been active in senior citizen groups. She is known to have taken butabarbital daily for many years and her daughter-in-law
states that since the patient had a seizure several years ago during attempts to lower the butabarbital dosage, she assumed that the
medication was for epilepsy. A bag of medications found at the patient*s home contains multivitamins, an acetaminophen/codeine
combination and naproxen. Uneventful surgical repair of the femur fracture is done the morning after admission. Postoperative
medication orders are for morphine, cephalothin, phenytoin and warfarin.

106. On the evening after the operation, the patient becomes combative, begins to hallucinate and has a brief, generalized
tonic-clonic seizure. Which of the following is the most likely cause for the change in her behavior?

(A) Barbiturate withdrawal


(B) Cerebral concussion
(C) Inadequate treatment of an underlying epileptic disorder
(D) Potentiation of morphine by phenytoin
(E) Warfarin side effect

The acute episode is appropriately managed and her therapeutic regimen is modified. At the time of transfer to an extended care
facility 6 days later, she is alert and cooperative. Her prothrombin time is stable at 1.3 times control values (INR=2.0). Two days
later, she experiences a transient episode of asymptomatic hematuria.

107. Which of the following is the most appropriate treatment?

(A) Administer fresh frozen plasma


(B) Administer intramuscular vitamin K
(C) Administer parenteral protamine
(D) Decrease the warfarin dosage
(E) Prescribe trimethoprim-sulfamethoxazole, orally

108. Which of the following is the most appropriate medication to relieve her arthritis pain while she continues warfarin therapy?

(A) Acetaminophen
(B) Aspirin
(C) Codeine
(D) Naproxen
(E) Phenylbutazone

END OF SET

74
109. A 50-year-old woman is in the recovery room following an uneventful cholecystectomy. Two hours after the procedure she
begins to have ventricular extra-systoles and her systolic blood pressure falls from 110 mm Hg to 90 mm Hg. Arterial blood
gas values while breathing room air are:
PO2 58 mm Hg
PCO2 52 mm Hg
pH 7.30

These signs are most likely the result of which of the following?

(A) Alveolar hypoventilation


(B) Occult hemorrhage
(C) Primary cardiac irritability and failure
(D) Primary hypoxemia resulting from anesthetic gases
(E) Pulmonary embolus

110. A 20-year-old woman who is 35 weeks pregnant with her first child is admitted to the hospital because of persistent
hypertension and 1+ protein on urinalysis obtained 36 hours ago. She is confined to bed awaiting further diagnostic studies
for preeclampsia. Her blood pressure is now 160/100 mm Hg. She is complaining of headaches, blurred vision and epigastric
pain. At this time it is most appropriate to conclude that this patient has which of the following?

(A) Chronic hypertension and requires antihypertensive therapy


(B) Chronic hypertension, but no antihypertensive therapy is necessary
(C) Mild preeclampsia and should continue bed rest pending further diagnostic studies
(D) Severe preeclampsia and requires immediate medical management and delivery
(E) Severe preeclampsia and should continue bed rest pending further diagnostic studies

111. A 52-year-old woman who has had low back pain for several years is admitted to the hospital because the pain has suddenly
worsened. Her current medications include oxycodone, amitriptyline, perphenazine, fluoxetine and trazodone. On physical
examination, the patient is 10% below her ideal body weight, pupils are constricted and skin turgor is poor. She seems
sluggish and her speech is slow. Results of neurologic examination and x-ray films of the lumbosacral spine are normal. If a
medication is responsible for her mental condition, the medication is most likely to be which of the following?

(A) Amitriptyline
(B) Fluoxetine
(C) Oxycodone
(D) Perphenazine
(E) Trazodone

75
Items 112-113

An obese 10-year-old boy with diabetes mellitus is admitted to the hospital because of severe ketoacidosis and cardiovascular
collapse. Initial management consists of cardiac monitoring and intravenous administration of fluids, electrolytes and insulin.
Because of the difficulty in obtaining satisfactory peripheral venous access, the left femoral vein is catheterized percutaneously. Six
hours later, his mental status is improved, blood pressure is 120/70 mm Hg and serum glucose concentration is 250 mg/dL.
Physical examination at that time shows a cold left foot with diminished pulses compared with those of the right foot.

112. The most likely explanation for the diminished pulses is

(A) paradoxical embolus from the femoral vein


(B) diabetic arteriopathy
(C) inadvertent injury of the femoral artery
(D) intense arteriolar constriction induced by hyperosmolality
(E) thrombosis of the catheterized femoral vein

113. The electrolyte deficit in this patient most likely to cause death is

(A) bicarbonate
(B) calcium
(C) chloride
(D) potassium
(E) sodium

END OF SET

114. A 22-year-old woman who has a 5-year history of ulcerative colitis is admitted to the hospital because of increasing
abdominal pain, diarrhea and hematochezia. Her disease is limited to the descending colon. Current therapy includes
sulfasalazine and corticosteroid enemas. While she is receiving parenteral corticosteroid therapy, she is discovered to be 8
weeks pregnant. The statement that should be emphasized in discussing this pregnancy with her is:

(A) Abortion should be considered since ulcerative colitis increases the incidence of premature births and congenital
anomalies
(B) Abortion should be considered because of the adverse effect of pregnancy on ulcerative colitis
(C) Abortion should be considered because of the teratogenic effects of her colitis therapy
(D) Occurrence of spontaneous abortions does not correlate with disease activity
(E) Sulfasalazine and corticosteroids are not known to be teratogenic

76
Items 115-116

You plan to discharge an 81-year-old woman in a few days following a 3-week stay in the hospital for repair of a fractured hip that
she sustained while gardening. She now ambulates with difficulty using a walker, but she is determined to become independent
again and to return to her own home. Her daughter, who is in the room with the patient, says, "I want to take Mother home with me
because I'm concerned that she could fall and break her hip again. Mom says she doesn't really want to leave her own home, but she
will do what is best." The daughter turns to her mother and says firmly, "Isn't that right, Mom?" The mother says, "Yes, I guess so,"
averting eye contact with both her daughter and you by looking down toward the floor.

115. The most appropriate response to the mother is:

(A) "Are you sure you want to go home with your daughter?"
(B) "How would you feel if you fell again and had another fracture?"
(C) "Is this really your decision or is it your daughter's?"
(D) "I would like to talk with you in private now."
(E) "You are lucky to have a daughter who wants to take care of you."

116. The step most likely to prevent another fall and a possible fracture in this patient is to

(A) advise her to walk only when accompanied by an adult


(B) ensure that she does not have orthostatic hypotension
(C) provide her with assistance for activities of daily living
(D) provide her with an electric wheelchair
(E) request a visiting nurse to assess the safety of her living environment

END OF SET

77
117.

A 57-year-old woman is admitted to the hospital for evaluation of nausea, vomiting, crampy abdominal pain and abdominal
distention. Her medical history includes cholecystectomy and appendectomy 5 years ago. X-ray film of the abdomen is
shown. The most likely cause of her symptoms is

(A) adhesive band


(B) femoral hernia
(C) gallstone ileus
(D) perforated diverticulum
(E) sigmoid colon carcinoma

118. A 56-year-old man is admitted to the hospital from the emergency department because of an acute myocardial infarction. He
has no identified risk factors and no history of heart disease. He had no major complications and is discharged after 8 days.
At discharge, the diagnostic test that should be scheduled for the 2-week follow-up visit in order to predict most accurately
his risk for another infarction is

(A) electrocardiogram
(B) exercise stress test
(C) 24-hour ambulatory electrocardiographic monitoring
(D) radionuclide ventriculography
(E) ultrasonography of the heart

78
119. A 57-year-old woman with metastatic breast cancer is admitted to the hospital for an investigational chemotherapy protocol.
Her cancer has thus far been unresponsive to treatment. At her request, the attending physician writes a "Do Not Resuscitate"
order. Two days later, at 1:30 AM, the patient has suddenly slumped over and the nurse is unable to obtain a pulse or blood
pressure reading. The patient has agonal respirations and then stops breathing. The first physician to arrive at the patient's
bedside should

(A) call the attending physician


(B) call the patient's husband
(C) pronounce the patient dead
(D) initiate cardiopulmonary resuscitation
(E) initiate intravenous chemical resuscitation

120. A 25-year-old woman has just given birth to a 3200-g (7-lb 1-oz) boy with an Apgar score of 9 at both 1 and 5 minutes. The
physical examination of the neonate is normal. The mother is HIV positive and received no prenatal care. She acquired HIV
infection from her husband who is also HIV positive. At the time of delivery her rapid plasma reagin test was 1:164 with a
positive fluorescent treponemal antibody test. When you visit the mother to discuss the medical care of her baby and to obtain
more history, you find her in tears. She asks you what the chances are of her baby being infected with HIV. It is most
appropriate to tell her which of the following?

(A) Since she has no symptoms of AIDS, there is a strong likelihood that her baby will not be infected
(B) Having untreated syphilis at delivery increases her baby's risk for being infected by 50%
(C) Her baby has at least a 50% chance of not being infected and will need to be followed closely since it is difficult to
make the diagnosis at birth
(D) If the results of ELISA and Western blot testing of the baby are positive, the baby is definitely infected
(E) Because she acquired her infection through a heterosexual transmission, there is only a 20% risk that her baby
is infected

NOTE: THIS IS THE END OF THE INPATIENT BLOCK.


ANY REMAINING TIME MAY BE USED TO CHECK ITEMS IN THIS BLOCK.

79
Block 4: Emergency Department
Items 121-144; Time - 30 minutes

Most patients in this setting are new to you, but occasionally you arrange to meet there with a known patient who has
telephoned you. Generally, patients encountered here are seeking urgent care. Also available to you are a full range of
social services, including rape crisis intervention, family support, and security assistance backed up by local police.

ALL ITEMS REQUIRE SELECTION OF ONE BEST CHOICE.

121. A 44-year-old woman comes to the emergency department because of chest pain, shortness of breath and fever. On physical
examination she is in mild respiratory distress. Vital signs are: temperature 37.8°C (100.0°F), pulse 100/min, respirations
24/min and blood pressure 100/60 mm Hg. Auscultation of the lungs discloses rhonchi at the right lung base posteriorly. The
chest pain is in the right anterior region and is pleuritic. Chest x-ray film discloses patchy infiltration of the right lower lobe.
The patient is diagnosed as having community-acquired pneumonia, and she is sent home with a prescription for
erythromycin and an analgesic medication. The patient returns 2 days later complaining of new onset of swelling of her right
leg and foot. Noninvasive vascular studies show a proximal deep venous thrombosis in the right leg, and ventilation-
perfusion lung scan is interpreted as high probability for a pulmonary embolus. The patient is given anticoagulation therapy
and recovers. The care this patient received initially is best characterized as which of the following?

(A) An error in diagnostic accuracy


(B) Legal negligence
(C) Malpractice
(D) A therapeutic misadventure
(E) Violation of the principle of nonmaleficence

122. A 46-year-old man with Marfan syndrome, aortic insufficiency and mitral regurgitation comes to the emergency department
because of severe substernal chest pain for the past 3 hours. He describes the pain as tearing in quality and radiating to the
neck. One week earlier he experienced similar, but less severe, chest pain and treated himself with aspirin. Which of the
following is the most likely underlying cause for his worsening symptoms?

(A) Acute bacterial endocarditis


(B) Acute myocardial infarction
(C) Dissection of the aorta
(D) Esophageal reflux with spasm
(E) Perforated peptic ulcer

123. A 15-year-old African-American girl comes to the emergency department because, she says, "something is sticking out of
my bottom since I had a bowel movement this morning." She denies previous episodes, although for more than 1 year she
has had occasional difficulty passing stools. She is not in pain at present, but she is afraid to move her bowels for fear that the
problem will get worse. In response to your questions, she tells you that she moved away from home more than a year ago
and her parents contribute nothing to her support. She has a 6-month-old child and lives with a 28-year-old female cousin.
She has never been married and does not work or attend school. She has no other symptoms. In order to follow the correct
procedure for treating a minor, which of the following is the most appropriate step prior to evaluating this patient's rectal
problem?

(A) Accept the girl's consent as sufficient


(B) Obtain a court order permitting evaluation
(C) Obtain the written consent of at least two licensed physicians
(D) Obtain written consent from at least one of her parents
(E) Obtain written consent from her 28-year-old cousin

80
124. A 29-year-old woman comes to the emergency department because of increasingly severe lower quadrant pain and nausea
for the past 2 days. She says that her menstrual period ended 6 days ago. She is sexually active and does not use any
contraception. Temperature is 38.3°C (101.0°F), orally. On physical examination, there is bilateral lower quadrant
tenderness with rebound and guarding. Pelvic examination shows leukorrhea at the cervical os and the uterus is tender to
palpation. The adnexal areas are tender but no masses are palpable. Which of the following is the most appropriate
diagnostic study?

(A) Cervical culture


(B) Culdocentesis
(C) Laparoscopy
(D) Serum β-subunit of human chorionic gonadotropin (β-hCG) concentration
(E) Ultrasonography of the pelvis

125. A 26-year-old man is brought to the emergency department by his family because he says that he is being followed by
gangsters and that they are going to kill him. Temperature is 37.8°C (100.0°F), pulse is 110/min and blood pressure is
160/95 mm Hg. His pupils are dilated. The remainder of the physical examination is normal. The family states that he has a
history of drug abuse. Which of the following drugs most likely caused this reaction?

(A) Alcohol
(B) Cocaine
(C) Diazepam
(D) Heroin
(E) Methaqualone

Items 126-127

A 35-year-old woman is seen in the emergency department because of the sudden onset of severe low back pain 12 hours earlier.
The pain began when she bent over to pick up her 2-year-old child. She has been unable to stay in bed because of the need to care
for her child. Low doses of ibuprofen have eased the discomfort slightly. Careful physical examination, including a neurologic
examination, is normal except for evidence of muscle spasm. She believes she has a herniated disc because 2 years ago her father
developed the sudden onset of back pain that required immediate surgery.

126. Which of the following is the most appropriate first step?

(A) Order CT scan of the lumbar spine


(B) Order MRI of the lumbar spine
(C) Order x-ray films of the lumbar spine
(D) Reassure her and treat with a nonsteroidal anti-inflammatory drug (NSAID)
(E) Request consultation with an orthopedic surgeon

127. She returns 3 weeks later because of persistent left lower leg pain and a weak left ankle-jerk reflex. In addition to pain
management, which of the following is the most appropriate step at this time?

(A) AP and lateral x-ray films of the lumbosacral spine


(B) CT scan of the lumbosacral spine
(C) Diskography
(D) MRI of the spine
(E) Observation

END OF SET

81
128. A 9-year-old boy is brought to the emergency department by his father because the boy is slightly lethargic and has labored
breathing. The father, who is a single parent, reports that the boy is "always thirsty" and "urinates a lot." The boy's pulse is
120/min, respirations are 32/min and blood pressure is 110/65 mm Hg. Laboratory studies show:

Serum
Na+ 132 mEq/L
K+ 4.1 mEq/L
Cl! 92 mEq/L
HCO3! 6.6 mEq/L
Creatinine 1.0 mg/dL
Glucose 850 mg/dL

The boy is treated with intravenous insulin and isotonic saline solution. Several hours later, he is improved and his serum
glucose concentration is 450 mg/dL. Which of the following is the most appropriate next step?

(A) Add glucose to this intravenous solution


(B) Add potassium to this intravenous solution
(C) Add sodium bicarbonate to this intravenous solution
(D) Begin treatment with intermediate-acting insulin
(E) Change the intravenous solution to hypotonic saline solution

129. A 28-year-old woman who is known to be HIV-positive comes to the emergency department because of increasing
headaches, right-sided weakness and disorientation for the past week. A major motor (grand mal) seizure occurs shortly after
admission. On physical examination following the seizure, vital signs are normal. There is no nuchal rigidity. Funduscopic
examination shows papilledema. There is also right hemiparesis and aphasia. Which of the following is the most likely
diagnosis?

(A) Meningioma
(B) Meningococcal meningitis
(C) Neurosyphilis
(D) Toxoplasmosis
(E) Tuberculous meningitis

130. A 38-year-old nurse comes to the emergency department after leaving work early because of a "horrible headache." She has
had a "cold" with sinus congestion for the past week, and yesterday she began taking an over-the-counter combination of
diphenhydramine and pseudoephedrine. She tells you she has a history of "migraines," multiple allergies, premenstrual
syndrome and depression, for which she takes phenelzine (a monoamine oxidase inhibitor). Vital signs are: temperature
37.2°C (99.0°F), pulse 90/min, respirations 16/min and blood pressure 210/118 mm Hg. Which of the following is the most
appropriate action?

(A) Administer meperidine, intramuscularly


(B) Administer phentolamine, intravenously
(C) Order CT scan of the head
(D) Order transillumination of the sinuses
(E) Prescribe oral oxycodone and nasal corticosteroids

82
Items 131-132

A 48-year-old Native American construction worker sustained a comminuted fracture of his left tibia and fibula 4 months ago. For
the past 3 months he has been in the rehabilitation unit with his leg fully immobilized. Three hours ago he suddenly developed
chest pain and shortness of breath, and he has just been brought to the emergency department for further evaluation. On
examination he describes an aching discomfort over the right superior anterior chest and the right scapula posteriorly. The family
history is strongly positive for heart disease.

131. In questioning the patient further, an important point in the history would be the relationship of the pain to which of the
following?

(A) Change of position


(B) Deep breathing
(C) Eating
(D) Swallowing
(E) Walking

132. The presence of a right pleural friction rub in this patient would suggest which of the following?

(A) Pericarditis
(B) Pneumonia
(C) Pneumothorax
(D) Pulmonary embolus with infarction
(E) Pulmonary embolus without infarction

END OF SET

133. A 57-year-old man is brought to the emergency department by the rescue squad after he was found lying unconscious in the
street. No other history is available. On physical examination he has a temperature of 40.0EC (104.0EF) and marked nuchal
rigidity. While awaiting the results of a lumbar puncture, which of the following is the most appropriate intravenous
pharmacotherapy?

(A) Ceftriaxone
(B) Ciprofloxacin
(C) Glucocorticoids
(D) Penicillin
(E) Ticarcillin

83
Items 134-136

A 5-year-old boy is brought to the emergency department by his parents because of a swollen pinna. The father says that the boy fell
off his tricycle in a playground. On physical examination the child's left ear is severely edematous and discolored. There is concern
for the child because this is his third emergency department visit in the past 6 months. When discussing these concerns and the
need for further evaluation with the parents, they become angry and want to take the child home now.

134. It would be most appropriate to

(A) arrange for the child to be admitted for evaluation and protection
(B) discharge the child to another relative or neighbor if available
(C) keep the child and initiate foster care placement
(D) send the child home with his parents, but notify the police of the situation
(E) send the child home and arrange for a social worker to make a family assessment on a home visit

135. This case should be reported to the child protective service agency

(A) if another licensed physician agrees with the current assessment and documents this in the chart
(B) if there is a pattern of repeated suspicious injury
(C) if there is proof that the parents inflicted the injury
(D) if the injury is judged to be life-threatening
(E) on the basis of the current suspicion alone

136. Even after informing the parents of the decision to report the case to the child protective services, they continue to deny
having hurt the child and are furious. If the diagnosis of child abuse is incorrect, the reporting physician is liable for

(A) civil charges


(B) criminal charges
(C) medical license revocation
(D) state medical society censure
(E) no damages or penalties

END OF SET

137. A 42-year-old Anglo-American man is brought to the emergency department by his same sex partner because of confusion,
diplopia and mild right arm weakness. On examination the patient is somewhat agitated and shows confusion for recent
events. There is decreased pupillary response on the left with some paresis of lateral gaze on the right. Temperature is
38.3°C (101.0°F). Peripheral leukocyte count is increased. The most appropriate next step in evaluation of his neurologic
signs and symptoms is

(A) bilateral carotid arteriography


(B) CT scan of the head
(C) electroencephalography
(D) lumbar puncture
(E) serum test for HIV antibodies

84
138.

A 60-year-old man comes to the emergency department after sustaining facial injuries in a fight. He is mouth breathing,
apparently due to his injuries, but he denies any respiratory problems. He is known to be alcohol- and drug-dependent. He
has smoked one to two packs of cigarettes per day for 35 years. There is dullness to percussion and rales over the right upper
lobe. Chest x-ray film shown is obtained. The most likely cause of the findings on this x-ray film is

(A) aspergillosis
(B) carcinoma
(C) pneumocystosis
(D) sarcoidosis
(E) tuberculosis

85
139.

A 38-year-old Hispanic bank executive comes to the emergency department because of the sudden onset of shortness of
breath, light-headedness, diaphoresis and weakness. He is afebrile. On auscultation of the lungs, bibasilar rales are heard.
Electrocardiogram is shown. The most likely diagnosis is

(A) acute pericarditis


(B) hyperventilation syndrome
(C) myocardial infarction
(D) pulmonary embolism
(E) spontaneous pneumothorax

140. A 62-year-old man strikes the steering wheel of his car during a low-speed automobile accident. In the emergency
department he is alert and his vital signs are normal. He complains of mild tenderness on sternal compression. Chest x-ray
film shows a widened mediastinum. The most appropriate first step is to order

(A) CT scan of the chest with contrast


(B) bronchoscopy
(C) a MUGA scan of the heart
(D) thoracentesis
(E) thoracic ultrasonography

86
141. A 44-year-old African-American construction worker comes to the emergency department because of excruciating left flank
pain that radiates to his left testicle. He describes the pain as coming in waves, and he says, "This is the worst pain I’ve had
in my life, and that includes closing my thumb in my truck door." On physical examination he is extremely restless and is in
obvious pain. Genitalia are normal. Abdominal examination is normal except for intermittent guarding with spasms of pain.
Plain x-ray film of the abdomen is normal. Urinalysis and urinary sediment are shown:

pH 6.5
Specific gravity 1.025
Glucose negative
Protein negative

The most appropriate diagnostic study is

(A) CT scan of the kidney


(B) culture of the urine
(C) determination of serum uric acid concentration
(D) CT scan of the abdomen
(E) measurement of 24-hour urinary calcium excretion

142. A 19-year-old woman comes to the emergency department because, she says, "I'm burning up." She is known to staff as an
intravenous drug user. On physical examination a systolic heart murmur is detected over the precordium. An expected
physical finding will be which of the following?

(A) Decreased intensity of S1


(B) Increased intensity of the murmur with deep inspiration
(C) Increased intensity of the murmur with forced expiration
(D) Positive Kussmaul sign (rise in jugular venous pulse with inspiration)
(E) Right-sided gallop

87
143. A 23-year-old man comes to the emergency department because of pain, swelling and erythema over the
metacarpophalangeal joint of the right long finger. Three days ago he struck an opponent in the mouth during an
altercation. On physical examination there is an abrasion over the dorsal surface of the joint, pain on passive motion of the
finger, and exquisite tenderness along the volar aspect of the finger and metacarpal. His temperature is 38.5°C (101.3°F).
X-ray films are normal. Which of the following is the most appropriate management?

(A) Splinting for 1 week followed by active range-of-motion exercises


(B) Splinting for 3 weeks followed by active range-of-motion exercises
(C) Splinting, oral antibiotic therapy and reevaluation in 24 hours
(D) Splinting and intravenous antibiotic therapy
(E) Surgical debridement and intravenous antibiotic therapy

144. A 25-year-old man is brought to the emergency department by his wife because he has been vomiting for the past 24 hours.
He has used prochlorperazine suppositories for relief of nausea and vomiting. He now has severe muscle spasms in his neck.
On physical examination there is sustained spasm of the sternocleidomastoid and trapezius muscles with twisting of his head
to the right. Which of the following is the most appropriate pharmacotherapy at this time?

(A) Chlorzoxazone
(B) Dantrolene
(C) Diazepam
(D) Diphenhydramine
(E) Methocarbamol

NOTE: THIS IS THE END OF THE EMERGENCY DEPARTMENT BLOCK.


ANY REMAINING TIME MAY BE USED TO CHECK ITEMS IN THIS BLOCK.

88
Answer Key for Step 3 Sample Questions

Block 1

1. A 9. B 17. A 25. B 33. A 41. C


2. C 10. B 18. E 26. A 34. E 42. B
3. A 11. E 19. A 27. E 35. B 43. B
4. E 12. D 20. A 28. C 36. E 44. A
5. A 13. B 21. A 29. B 37. A 45. C
6. E 14. B 22. C 30. C 38. C 46. C
7. A 15. C 23. B 31. C 39. B 47. B
8. A 16. B 24. A 32. A 40. D 48. D

Block 2

49. E 57. C 65. C 73. D 81. E 89. A


50. C 58. C 66. D 74. E 82. A 90. C
51. D 59. A 67. B 75. E 83. E 91. A
52. D 60. B 68. A 76. E 84. B 92. E
53. D 61. A 69. E 77. E 85. D 93. D
54. D 62. E 70. B 78. C 86. B 94. E
55. D 63. C 71. D 79. A 87. C 95. B
56. B 64. B 72. A 80. C 88. C 96. D

Block 3

97. A 101. A 105. B 109. A 113. D 117. A


98. D 102. E 106. A 110. D 114. E 118. B
99. A 103. C 107. D 111. C 115. D 119. C
100. D 104. A 108. A 112. C 116. E 120. C

Block 4

121. A 125. B 129. D 133. A 137. B 141. D


122. C 126. D 130. B 134. A 138. E 142. B
123. A 127. D 131. B 135. E 139. C 143. E
124. A 128. B 132. D 136. E 140. A 144. D

89

You might also like